Vous êtes sur la page 1sur 79

I.

Sentence Correction Introduction

Of all the topics you must study to prepare for the GMAT, there are two in particular that will bring great
long-term benefits: the AWA Essay and the Sentence Correction sections. Effective writing is a vital
part of business communication, and the skills you acquire in preparation for these sections will be keep
working for you long after test day.

About 14 of the 41 Verbal section questions are Sentence Correction.

The directions for these questions look like this:

Directions: The following questions consist of sentences that are either partly or entirely underlined.
Below each sentence are five versions of the underlined portion of the sentence. Choice (A) is a copy of
the original version. The four other answer choices change the underlined portion of the sentence. Read
the sentence and the five choices carefully and select the best version.

Choose answers according to the norms of standard written English for grammar, word choice, and
sentence construction. Your selected answer should express the intended meaning of the original
sentence as clearly and precisely as possible, while avoiding ambiguous, awkward, or unnecessarily
wordy constructions.

To help you learn this the 800score approach is to focus on the most common error types. We rely
heavily on graphic diagrams to help teach grammar concepts. Every sentence has its own structure,
and understanding the engineering of the parts of a sentence is key to mastering grammar on the
GMAT.

The chapter is divided into five parts:

1. Sentence Correction Tips


A few basic tips to keep in mind.

2. Glossary of Grammar Definitions

3. Three-Step Method for Sentence Correction Questions


This section provides you with a clear, step-by-step method for tackling all Sentence Correction
questions.

4. Seven Types of Errors in the Sentence Correction Section


This section provides you with an overview of the seven most common grammar mistakes
found in the Sentence Correction section. You will learn specific skills for handling individual
questions.

5. Sample Questions
Timed online questions to simulate actual GMAT questions.

II. Sentence Correction Tips


1. GMAT grammar adheres to the rules of "Standard Written English"

"Standard Written English" refers to the grammar rules that you find in grammar books and in formal
writing. Since proper written English often differs from spoken English, the best answer will not always
be the one that sounds the best. You cannot rely on your ear alone; you must become familiar with the
grammar rules of written English.

2. The GMAT tests a limited number of grammar rules.

English grammar contains hundreds of very specific rules. The GMAT only tests a few of these, so
devote your energies to mastering the rules that most frequently come up.

3. Grammar is key - but style is important, too

The best answer must be clear, without unnecessary redundancy, and with proper punctuation. Idioms
must be used correctly. Style is a secondary concern. Look for grammar errors first, and then check for
errors in style.

4. Don't change the meaning of the sentence

In the sentence correction section, you'll sometimes find two answer choices that are equally correct in
terms of grammar and style conventions. When this happens, choose the answer that best maintains
the meaning of the original sentence. The correct answer will never significantly alter the original
meaning.

5. Incorrect answer choices are incorrect

Sentence Correction answer choices are variations on the correct answer. Incorrect answers will almost
always be identifiable as such. Even if an answer choice sounds funny, if you can't find a definite error,
then don't rush to eliminate it.

III. Glossary of Grammatical Definitions

This is basic introductory information if you have a strong background in English, feel free
to skim or skip this section.

active voice in which the person or thing performing the action is the
subject of the verb

John throws the pencil.

adjective modifies a noun, pronoun, or other expression that does the


work of a noun (called a nominal)

It was a happy coincidence.


modifies a verb, adjective, or other adverb
adverb
The detective paced slowly around the room.

word (a, an, the) that specifies or confines the meaning of a


article noun

Definite Article: The soldier died bravely.

Indefinite Article: A soldier never truly returns home.

in a sentence, a group of words that contains both a subject


clause and a predicate

I couldn't believe that Barbara said those things.

joins two or more words, phrases, clauses, or sentences


conjunction
Sue and Sally have never been late; they are always on time.

indicates a group of persons, things, or animals treated as a


collective noun single entity

The fleet of ships arrives too late.

A chorus of angels quivers in her soul.

pair of words which, separated from each other in a


correlative sentence, act as a conjunction (joining two or more words,
conjunction phrases, or sentences)

Either you are coming with me, or we will never see each
other again.

noun formed from a verb, usually by adding -ing to the end


gerund
Running to catch a train can be very dangerous.

idiom word, or expression comprising several words, the meaning


of which extends beyond the usual meanings of the individual
words
Chocolate tastes as good as ice cream.

The candidate claims to support tax cuts, in contrast to his


prior statements.

Neither Tom nor Sam has the necessary skills to finish the
job.

pronoun that does not stand in for any particular noun, but
impersonal instead refers to "people in general" or fulfills the sentence's
pronoun syntactical need for a pronoun

One must pay close attention to a test's instructions.

It must be said.

dictionary form of a verb; in English, most often appears as


infinitive "to ___ " ("to eat", "to run")

To sleep, perchance to dream, ay there's the rub.

word, phrase, or clause that provides extra information about


modifier another word, phrase, or clause

The soft pillow made up a bit for the hard bed.

verb form that indicates the speaker's position on the


mood factuality of the sentence; indicates if action/condition is true
or unlikely, or if the speaker is giving a command

Indicative Mood:(statement of fact )


Harry spends all of his money on comic books.

Imperative mood: (command or instruction) Spend all of your


money on comic books!

Subjunctive mood: (indicates probability or objective


possibility)
I wish Paul were not spending his money so recklessly.

indicates a person, place, or thing


noun
John ate pizza at the cafe with his friends.

in a sentence, the noun or noun phrase that receives, or is


object otherwise affected by, the action specified by the verb

Geronimo ran to the cliff.

voice in which the person or thing performing the action is the


passive object of the verb

The batter was hit by the pitch.

phrase in a sentence, a group of words that contains either a subject


or a predicate and lacks the other

Noun Phrase: the mouse in the trap

Prepositional Phrase: under the full moon

Verb Phrase: runs around and around

Adjectival Phrase: good as gold

Adverbial Phrase: happily oblivious

noun or pronoun indicating multiple persons, places, or things


plural
Cows don't like sheep.

also the form of the verb (especially in the present tense) that
agrees with multiple persons, places, or things

Six cats are asleep on the rug.

pronoun or adjective indicating possession


possessive
Lucys book is over there.
(The proper noun Lucy is now used as a possessive adjective
Lucys)

That book over there is hers.


(Possessive pronoun)
part of a sentence or clause that, as a whole, modifies the
predicate subject; includes the verb, the object/s, or phrases presided
over by the verb

Ricky reads.

Ricky reads the newspaper.

Ricky reads the newspaper to his grandfather.

preposition shows relationship between words, phrases, or clauses

The man from Brazil had never seen snow.

The tax collector tapped on the door.

stands in for a noun or noun phrase


pronoun
John just meant to scare the boys. He made a terrible
mistake, though.

noun indicating a specific person, place, or thing


proper noun
The Angkor Vat Temple in Cambodia is one of Cassandra's
favorite works of architecture.

pronoun that connects a subordinate clause to the rest of the


relative pronoun sentence

Harry is the boy who won the race.

Harry is the boy that Julie had a crush on.

group of words that contains a subject and a predicate, and is


sentence able to stand on its own

This is a sentence.
singular noun or pronoun indicating one person, place, or thing

The cow doesn't like the sheep.

also the form of the verb (especially in the present tense) that
agrees with one person, place, or thing

This gun only shoots blanks.

in a sentence, the noun or noun phrase that performs the


subject action indicated by the verb; or which is explained or
described by the verb

The ship sailed through the night storm.

represents an action or state of being


verb
We all know this already.

voice set of verb forms indicating the relationship between the


subject and the action or condition expressed by the verb

The big fish swallowed Jonah.

Jonah was swallowed by the big fish.

IV. Sentence Correction Three-Step Method

A Sentence Correction question looks like this:

1. When Charlene goes to the park, she likes to run, swim, and to play basketball.

A. she likes to run, swim, and to play basketball


B. she likes to run, swim, and play basketball
C. she likes running, to swim, and to play basketball
D. she likes running, swimming, and to play basketball
E. she likes all of the following, to run, swim, and to play basketball

You are given a sentence with one section underlined, and five answer choices. The underlined
portion is reproduced five different ways in the answer choices - choice A will always be identical to the
underlined portion of the sentence. Your task is to find the answer choice which is most grammatically
correct according to the rules of Standard Written English. Sometimes more than one answer choice will
appear to be free of grammatical errors. This is not a mistake. Style conventions must be taken into
consideration as well. When this occurs, you must look for the answer that is clearly expressed and
concise.

800score Three-Step Method to Sentence Correction questions:

1.

Read the entire sentence. Do not simply read the underlined part of the sentence, because
context may be important in determining the correct answer. As mentioned above, choice A
will always be a copy of the original underlined part of the sentence.

Don't worry about spelling, capitalization, or punctuation; they are not covered in Sentence
Correction questions. If you do find an error in the underlined portion, or if you're not sure,
proceed to step two.

2.

The GMAT tests only a limited number of grammar error types. After you've read the
sentence, look for clues indicating which grammar rule the question is testing. These
grammar rules, and the clues to look for, will be covered in more detail in the next section.

Keep an eye out for:


Agreement Issues: Look for pronouns, verbs, and nouns: do they agree?
Modifiers: Look for introductory phrases set off by a comma: is the modifier used correctly?
Parallels: Look for commas separating words in a list, as well as expressions such as not
only...but also, both...and, either...or, neither...nor: is everything parallel?

3.

After you've dissected the question, compare answer choices and note the differences. Look
for the answer choice that preserves the meaning of the original sentence and fixes its
errors without creating any new ones. Eliminate answer choices with grammar errors.
Now that you have a method for approaching the questions, it's time to move on to specifics: how to
recognize and correct the seven common grammar errors found in the GMAT.

V. Seven Error Types

The GMAT tests only a limited number of grammar error types. Therefore, you only need to learn a
few of grammar rules you don't need to master every grammatical and stylistic rule of Standard
Written English to do well on the GMAT.

Seven Types of Errors


in the Sentence Correction Section

1. Subject-Verb Agreement
2. Modifiers
3. Parallelism
4. Pronoun Agreement
5. Verb Time Sequences
6. Comparisons
7. Idioms

1-A. Subject-Verb Agreement: Introduction

Subject-Verb Agreement

A. Introduction
B. Subject / Verb Separation
C. Collective Nouns
D. Plural / Singular
E. Neither / Either
F. Or / Nor
G. Subject / Verb / Object
H. Quantity Words
I. Sample Questions

Subjects and verbs must agree. The subject of a sentence is the noun to which the verb in the
sentence's main clause refers, and the two must always agree in number: singular subjects must be
paired with singular verbs; and plural subjects, with plural verbs. Though it may sound simple, the GMAT
uses tricky constructions and phrasings that make these questions seem far more complicated than
they actually are.
Test writers will try to fool you by writing unusual phrases that make it difficult to tell if the subject is
singular or plural. Below, you'll find a list of rules and tips for subject-verb agreement that will assist you
in making sense of the GMAT's intentionally confusing questions.

1. What "Agreement" Means in English Grammar

"Agreement" is one of the fundamental rules of English grammar. Essentially, the rule dictates that
corresponding parts of a sentence must match "agree with" one another. What exactly does this
mean? If you've ever studied a foreign language, like French or Italian, then you're already familiar with
the concept of agreement: when you change the form of a word to make it fit properly into the sentence,
like making the ending of an adjective masculine or feminine depending on whether the noun it modifies
is masculine or feminine - that's agreement. Agreement allows us to show who's doing what in a
sentence in a clear and unconfusing manner. Without such a method of clarification of indicating which
adjectives in a sentence modify which nouns, or which nouns are governed by which verbs clear
expression, in writing and speech alike, would be very difficult.

Because English is structured differently than French or Italian (or Spanish or Polish or Latin, etc., etc.),
the changes you have to make in order to adhere to the rules of agreement will be different. But the
basic concept of assuring a "match in form" between corresponding parts of a sentence is the same.

2. Kinds of Agreement

In the English language, agreement applies to several different kinds of pairings: nouns and their verbs;
pronouns and their verbs; adjectives and the nouns they modify; prepositions and the verb(s) they
govern. Depending on the nature of the pairing, agreement may be required in terms of tense, number,
or case. Tense and number apply to verbs; number applies to verbs, nouns, and pronouns; and case
applies to nouns, pronouns, and modifiers:

Tense: indicates when an action happened, is happening, or will happen

Number: indicates "how many" - singular (one) or plural (more than one)

Case: indicates role of noun or pronoun in sentence. In English, nouns only change form for the

possessive case, while pronouns have different forms for the possessive, subjective, and objective
cases.

Agreement requires that corresponding parts of a sentence match in as many of these ways as
possible: for example, a noun and verb can agree in terms of number (singular or plural) only, while a
noun and a pronoun can agree in terms of both number (singular and plural) and case (possessive or
otherwise). For native English speakers, agreement is for the most part instinctive, presenting little
difficulty in simple constructions: singular nouns agree with singular verbs, plural nouns with plural
verbs, and so forth.

But more complex sentences - those containing several nouns, verbs, and/or pronouns can
sometimes make even the simplest applications of agreement confusing.

3. Agreement in Action

In order to apply agreement rules, you must be able to recognize when agreement is required: what
parts of the sentence are "corresponding parts"?
Because a sentence can contain more than one noun and verb, subject-verb agreement helps to clarify
which noun is the subject, and by which verb it is governed. Refer to the main chapter for help on
recognizing and correcting subject-verb agreement errors on the GMAT.

1-B. Subject-Verb Agreement: Subject / Verb Separation

A subject and verb may be separated by an accompanying phrase without changing the
agreement.

The child, together with his grandmother and his parents, is going to the beach.

This sentence is grammatically correct. When a phrase sandwiched by commas comes between a
subject and a verb, the subject and verb must still agree, even if the sandwiched phrase contains other
nouns. The accompanying phrase "his grandmother and his parents" only provides extra information
and does not alter in any way the grammatical relationship between the subject (the child) and the verb
(is going).

Pay special attention to who or what is doing the action indicated by the verb, and make sure it agrees
with the verb. Ignore everything else.

Here's any easy way to handle this kind of "sandwich" agreement question. Take a look at the following
sentence and decide whether it is correct or incorrect:

Frank, accompanied by his students, were at the studio.

There are three nouns in this sentence, and two verbs. To clarify which of the three nouns is the
subject of the sentence, and with which of the two verbs the subject should agree, cross out
everything inside the commas, like so:

Two nouns remain: the subject is the noun in front of the crossed-out sandwich ("Frank"). The verb
we're looking for, the "main-clause" verb, is the only remaining verb in the sentence ("were").

To simplify the task of comparing the newly-identified subject and its governing verb, we'll next erase the
crossed-out sandwich. We're left with the following:
The subject of the sentence is now right next to its governing verb. But does this subject-verb
combination "Frank were" make sense? No. Frank is only one person signifying singularity, not
plurality - and so our governing verb should also be singular.

The plural verb "were" has been changed to the singular verb "was." This final version pairs a singular
noun with a singular verb, which corrects the original agreement error (a singular noun with a plural
verb).

It would be a good idea to practice this technique on your own before test day, because you often won't
have the time or space to work out each step at length. Once you have it down, this "cross-out" method
is by far the quickest and easiest way to identify agreement errors. Just by crossing out the section
inside the commas in this example, we were able to isolate, and then correct, the subject-verb
relationship: since Frank, a singular proper noun, is the subject of the sentence, not his students, a
singular, not plural, noun is required: Frank was at the studio.

Incorrect: Frank, accompanied by his students, were at the studio.

Correct: Frank, accompanied by his students, was at the studio.

Check for agreement in every question you see, and be aware of the different ways the error can
pop up. So how should you handle or even identify a subject-verb agreement error without obvious
isolating commas?

Here's are two types of filler phrases you will often see:

A. "Of" Phrases

A sentence will often begin with a noun, immediately followed by a group of words beginning with "of"
that includes another noun. When two or more nouns precede a verb, it can sometimes be hard to tell
which noun the verb should agree with. But that's where the concept of additive phrases can help us. In
most cases, "of" phrases are added just to complicate the sentence, and can be crossed out, leaving us
with a simple noun-verb agreement question.

Look at this sentence:

The goal of the architects are to create the most stunning and functional building in the city.
Does the verb agree with the subject? It's difficult to say at first glance, because we don't know yet what
the subject is. TWO nouns precede the verb: which is the subject?

If the plural noun "architects" is the subject, then the plural verb "are" is in fact correct. But if the singular
noun "goal" is the subject, then the plural noun "are" is incorrect.

To find the subject, cross out all the words between the first noun and the verb: this is the "of" phrase. As
with the sandwich questions, the best way to clarify agreement issues is to actually cross out the "filler"
(the additive phrase):

Once the filler phrase is crossed out, we can see that the plural verb "are" is not correct, because "goal,"
a singular noun, is the subject of the sentence. The correct verb is the singular "is":

Thus, even though the plural noun architects is closer to the verb than the singular noun goal, it holds
no weight in the sentence (in terms of agreement) simply because of its placement within the filler
phrase. The singular noun goal is the subject of the sentence, and a singular noun requires a singular
verb: is.

Incorrect: The goal of the architects are to create the most stunning and functional building in the

city.

Correct: The goal of the architects is to create the most stunning and functional building in the city.

B. "For" Phrases
"For phrases" are similar to "of phrases" add extra information to a sentence, while their contents does
not affect noun-verb agreement in the main part of the sentence.
Look at these two sentences. Which is correct?

The book I bought for my students tells the story of a Russian immigrant's experience in the United
States.

The book I bought for my students tell the story of a Russian immigrant's experience in the United
States.

The portion of the sentence we're concerned with contains two verbs in addition to there being three
possible subjects: two nouns, and one pronoun.

How do you know which noun is the subject, and which verb is the important verb? First, cross out the
"for" phrase:

The book I bought for my students tells the story of a Russian immigrant's experience in the United
States.

The book I bought for my students tell the story of a Russian immigrant's experience in the United
States.

That eliminates one noun, and leaves us with a noun and a pronoun vying for subject, and two verbs.
Next, cross out any cohered noun- (or pronoun-)verb groups:

The book I bought tells the story of a Russian immigrant's experience in the United States.

The book I bought tell the story of a Russian immigrant's experience in the United States.

You can also cross this out with the "for" phrase, if it's easier for you. Remember, that in additive phrase
questions the subject and its verb will never be right next to one another: the function of the additive
phrase is to separate them in order to confuse you. So if you've already eliminated the "for" or "to"
phrase and still have other nouns and verbs remaining, eliminate any noun-verb or pronoun-verb groups
that are right next to one another. The remaining noun and verb are your targets.

The subject, the singular noun "book," requires a singular verb.


Correct: The book I bought for my students tells the story of a Russian immigrant's experience in

the United States.

Incorrect: The book I bought for my students tell the story of a Russian immigrant's experience in

the United States.

By using the same method as we used for the "sandwich" questions, we were able to isolate, analyze,
and eventually correct the subject-verb relationship. Once you identify a phrase as a "filler" phrase,
you've made the question as simple as a "sandwich". All that's left to do is cross out, analyze, and
correct if necessary.

1-C. Collective Nouns

Collective nouns, such as family, majority, audience, and committee are singular when they act
in a collective fashion or represent one group. They are plural when the members of the collective
body act as individuals. Collective nouns will usually be singular in Sentence Correction
sentences. The difficulty of these questions lies in identifying a noun as a collective noun.

A majority of the shareholders wants the merger.

These nouns usually look plural, but are in fact singular. Confused? If you're having trouble determining
singularity or plurality, it might be helpful to visualize what's actually going on in the sentence. Ask
yourself these questions:

Is the sentence talking about something that acts as a singular entity?

Or, is it talking about the individual elements within that entity?

In the sentence above we are presented with the noun majority. The majority of shareholders likely
contains several shareholders; however, they are only spoken of as a group, not as individuals. There is
no indication that the sentence is referring to the individuals within the majority even though it
comprises several people, the majority acts as one as a singular entity and therefore requires a
singular verb, wants.

Here is another example:

The flock of birds is flying south.

This sentence presents another ambiguous noun flock followed by a plural noun, birds. Again, the
confusing noun is referred to as a singular group: even though a flock comprises many birds, we're not
talking about each bird's direction of flight, but the direction of the flock as a whole. And because the
flock as a whole is singular, it therefore requires a singular verb to accompany it: the singular verb is, not
the plural verb are.

Here is an example of a collective noun that requires a plural verb. Even though you will not see
this very often on the GMAT, it's helpful to illustrate the importance of reading the entire sentence and
visualizing what it describes every time you come across a confusing noun.

The sentence above describes the fighting that occurs between the individual members of the team.
Because "team" refers to several individual members, it is a plural noun, and therefore requires a plural
verb - "are" - as a result.

The key to these questions is simplicity:

1. recognize the collective noun

2. visualize what's going on in the sentence to make sure it is a collective noun

3. proceed

These questions are included in the GMAT not because they are especially difficult, but because test
writers expect most students to be unfamiliar with the rules governing collective nouns. But if you know
to look out for those tricky collective nouns, then you have no reason to worry, because you're already
ahead of the game.

List of Common Collective Nouns

army clergy government

audience council jury

band (musical band) crowd majority

board (political) department minority

cabinet (political) enemy public

choir group school

class herd senate

committee faculty society

company family

corporation team
1-D. Subject-Verb Agreement: Plural / Singular

In this example, we see a list of three names. Because these names Ted, John, I - are separated
by the word "and", the plural form of the verb is used, according to the rule stated above. Don't
over-think this one. It's an extremely straightforward grammatical construction: the subject is plural
because it refers to more than one person (or place, or thing, or event), and plural subjects require
plural verbs. So we use a plural verb.

In this example, we see another list, of two names. But this time the verb is singular, not plural. Why?
Because the names are separated by the word "nor." And according to the rule above, when a list of
things is separated by the word "nor," the singular form of the verb is used.

This construction is, as you can probably tell, the more complicated of the two. At first glance, it looks
very much like the and construction (though the meaning is the opposite). But, in fact, the two
constructions are rather different. In the first sentence, three people are going. In the second sentence,
neither one of two individuals, Ted and John, is going. Since, in the second sentence, the two people are
referred to individually, as separate entities, it wouldn't make sense to use a verb that refers to them as
one unit.

The way the second sentence is constructed is rather like using the term "no one." Would you say, no
one are going? Or, no one is going? You'd say the latter. How could "no one" be plural? Do the same
with "nor" and "or" constructions.

Here are some more words whose grammatical numbers (singular or plural) are commonly confused.

A. Pronouns

A. The following pronouns are always singular:

anyone everything something


anybody whatever no one

anything whoever/whomever nobody

everyone someone nothing

everybody somebody

Many of the words in this category can be broken down in a way that illustrates their essentially singular
nature:

B. The following pronouns are always plural:

both many several

few others

Memorize these.

C. The following indefinite pronouns could be either singular or plural


depending on context:

some none most

any all

''Depending on context" means that, for the pronouns in this list, you can't depend on memorization.
Look at these examples:

Some of the bananas are brown.

Some of the banana is brown.

Both sentences are correct. Why does the first require a plural verb, and the second a singular?
Because, in the first sentence, some refers to several distinct objects:
If we have, say, ten bananas, then some of the bananas means either two bananas, or three bananas,
or four or five or six in any case, many individual bananas. But in the second sentence, some refers
to part of one object:

One part of one banana is brown. In this sentence, some means "part" which is singular.

This trick works for the words some, all, any, and most. For any, the verb will usually appear before the
noun, but the same principle applies: if any is followed by a singular noun, use a singular verb; if it's
followed by a plural noun, use a plural verb.

It's exactly the same concept: the direction (the arrow in these diagrams) is just reversed. The trick
applies equally well for some, all, any, and most.

The pronoun none follows slightly different rules. Look at these sentences:

None of the ice cream was left over.


None of my friends are going to a play tonight.

None of the inmates was given a fair trial.

See something strange? The first and second sentences look fine, with a singular noun followed by a
singular verb and a plural noun followed by a plural verb. But the third sentence contains a plural noun
and a singular verb. How could this be?

Unlike agreement for the pronouns all or some, agreement for none isn't always determined by the noun
following it. The word none can be used to mean either "not any" or "not one," and sometimes,
only context, or the writer's intended emphasis, can determine which use is better. Fortunately,
however, in most cases probably all cases on the GMAT - its meaning in the sentence, and the verb it
requires, can be derived the same way it is for the other pronouns: by referring to the noun immediately
following it. If the noun is plural, the verb is plural as well; if the noun is singular, the verb is singular.
Let's look again at the first two sentences:

The noun following none ice cream is singular, and so the verb should be singular as well. It is also
helpful to note that none is most often singular when it means no part of as in, none of the ice cream,
or none of the chicken, or none of the baseball field.

None of the ice cream was left over

can become

None of it was left over

Here's a case in which none is plural:

Again, fairly straightforward: the noun following none is plural, so the verb is plural as well. When plural,
none means "not any":

Not any of my friends are going to a play tonight.

The alternative is for none to mean "not one," which carries essentially the same emphasis as "not a
single one." So unless the friends attend a play every night, there's no reason to say that not a single
one of my friends is going.

The second sentence, however, does require such emphasis:


None of the inmates was given a fair trial.

When you come across a confusing sentence like this, in which the noun is plural, but you're not sure
whether the verb should be too, ask yourself this question: would none be better replaced with not any,
or not one? If not any, use a plural verb; if not one, use a singular. Here, there is reason to emphasize
that "not even one" of the inmates was treated fairly, so the verb should be singular.

Not a single one of the inmates was given a fair trial.

Don't worry too much about sentences like this last example since the intention behind the use of
none in such cases can generally only be determined by context, and is often subtle, the GMAT is very
unlikely to include them in the Sentence Correction section.

None is singular when: None is plural when:


The noun following "none" is singular The noun following "none" is plural

OR OR

None means "no part of" None means "not any"

None means" not one" or "not a single one"

1-E. Subject-Verb Agreement: Neither /


Either

Neither and either always take singular verbs when acting as the subject of a sentence.

Here we have an example of a sentence in which the word "neither," not the plural noun "rosebushes,"
is the subject of the sentence. As per the stated rules above, "Neither" takes a singular verb when it acts
as the subject of the sentence. Therefore, the singular verb "is" is correct. The sentence requires no
alterations.

.
Here we have a similar construction: in this sentence, the word "either" acts as the subject, and
therefore requires a singular noun. If you're at all confused, a helpful tip is to think of the word
"either" as an abbreviation of the phrase "either one". Once you do so, it's easier to see that the
phrase "of us" is just filler (after which you can, if you like, cross it out). The subject of the sentence is
"either (one)", and so this sentence therefore requires a singular verb: "is".

1-F. Subject-Verb Agreement: Or / Nor

Neither/nor and either/or are a special case. If two subjects are joined by or or nor, the verb
should agree with the subject that is closer to it.

If the conjunction nor appears in a sentence with neither; or the conjunction or with either, then
the "neither/either" rule as stated above no longer applies. That is, if you see "neither" followed by
"nor," or "either" followed by "or," you can't automatically assume that the verb should be singular, as we
did in the last section. "Neithernor" and "eitheror" means, at least in the world of GMAT grammar,
"be careful."

In these constructions, "neither" and "either" are no longer the subjects of their sentences. Instead, they
function as conjunctions, working in pairs with "nor" and "or" to join two other subjects in the sentence.
When this occurs, the verb agrees with whichever subject is closer to it.

This "neithernor" sentence contains two subjects: "supervisor," and "staff members." (Why is "client"
not a subject too? Because in these situations, the subjects are the two nouns immediately following the
words "neither" and "nor.") Since the latter subject, "staff members," is plural, we therefore need a plural
verb, too. The plural verb "were" is correct.

This example is identical, grammatically, to the one above, except that the correlative conjunction joining
the subjects is "either/or." The verb must therefore agree with the subject closest to it, which is "child," a
singular noun. The proper verb form is the singular "is."

Remember to apply this rule only when both items of the pairs "neither/nor" and "either/or" are
present in the sentence.

1-G. Subject-Verb Agreement: Subject / Verb / Object


Be careful to choose the right subject in sentences in which the verb precedes the subject.

Sometimes, a sentence is so chock-full of nouns and pronouns that identifying the subject can feel like a
task of monumental proportions. To approach this, let's talk about word order. Because nouns,
pronouns, and expressions that do the job of nouns (these are known as nominals) can function as
subjects or objects, we tend to rely on their placement in the sentence to determine which nominal is
serving which purpose. (In the rest of this section, we will use noun as shorthand for all types of
nominal.) Basic sentences follow the pattern Subject Verb Object.

Here is an example:

This sentence is one of the most straightforward you'll encounter. The first noun in the sentence, dog, is
followed by an active verb, ate, and since we know that the dog is performing the action indicated by
the verb, dog is therefore the subject of the sentence. The only remaining noun, homework, is
therefore the object.

Some sentences, however, will unfortunately stray from this simple pattern. When all nouns in the
sentence follow the verb, it can sometimes be very difficult to figure out which of those nouns is the
subject. What should you do in those situations?

Let's look at an example:

Incorrect: There is many reasons why I can't help you.

This sentence contains two verb constructions ( is and cant help ) and three nouns/pronouns (
reasons, I, and you). Sandwiched between the first and second verbs ( is , can't help ) are a noun and a
pronoun ( reasons, I); another pronoun ( you) follows the second verb. If we look carefully at the
sentence, we might notice that the clause I can't help you follows the traditional pattern, such that I
would be the subject. We may therefore be tempted to decide that I is the subject of the sentence as a
whole.

Correct: There are many reasons why I can't help you.


However, why I can't help you is in fact a subordinate, or dependent, clause. We know this because it
begins with why, and it means that the entire phrase functions as a direct object of the other part of the
sentence. We can therefore remove any nouns inside of the subordinate clause from our search for the
subject. And if we do this, lo and behold, we see that there's only one noun left. The subject is the only
noun that exists outside of the subordinate clause: the plural noun reasons. A plural subject requires a
plural verb, and so the plural verb are is correct.

A quick summary of how to recognize subject-verb agreement errors. Look


for:

A subject and verb separated by superfluous nouns ("the sandwich")

Collective nouns like majority, audience, family

Other confusing nouns; for example: data/datum (data, often misused as a

singular noun, is the plural of datum)


Separation by conjunctions like and, nor, neither

1-H. Subject-Verb Agreement: Quantity Words

The phrase the number of requires a singular verb. The phrase a number of
requires a plural verb. It may make it easier to distinguish them by
understanding that the number of is a (singular) noun phrase that can be the
subject of a sentence, while a number of is an adjectival phrase meaning
several.

The number of frogs in the pond is twice the number of fish.

A number of protestors are gathering outside the capitol building today.

When you see either phrase the number of , or a number of - disregard the singularity or plurality of
the noun following it. If you're having a hard time remembering to do so, try crossing out all information
between the "number" phrase and the first verb.
The noun following the number of does not impact the verb because the subject of the sentence is
number, which is singular. The noun following a number of will always be plural, because a number of
means several.

1-I. Subject-Verb Agreement: Subject / Verb / Object

1. The President of Costa Rica, along with two vice-presidents, are elected for a four-year term by the
people.

a) are elected for a four-year term by the people.


b) are elected, by the people, for a four-year term.
c) is elected for a four-year term by the people.
d) are elected for four-year terms by the people.
e) is elected for four-year terms by the people.

The President of Costa Rica, along with two vice-presidents, are elected for a four-year term by the
people.

Grammar issue presented:


subject/verb agreement: subject/verb separation
along with two vice-presidents separates the noun 'the president of Costa Rica' (singular) from the verb
are (plural)

a) are elected for a four-year term by the people.


Subject / verb agreement? NO (president / are)

b) are elected, by the people, for a four-year term.


Subject / verb agreement? NO (president / are)

c) is elected for a four-year term by the people.


Subject / verb agreement? YES (president / is)
Additional errors? none

d) are elected for four-year terms by the people.


Subject / verb agreement? NO (president / are)
e) is elected for four-year terms by the people.
Subject / verb agreement? YES (president / is)
Additional errors? Change in meaning! : four year terms

(C) shows the correct subject-verb agreement; no additional errors are created.

2. The Egyptian Museum in Cairo, which contains 120,000 objects from prehistoric times through the
Greco-Roman period, are home to one of the most impressive collections of ancient Egyptian artifacts.

a) which contains 120,000 objects from prehistoric times through the Greco-Roman period, are home to
b) which contain 120,000 objects from prehistoric times through the Greco-Roman period, are home to
c) containing 120,000 objects from prehistoric times through the Greco-Roman period, are home to
d) which is containing 120,000 objects from prehistoric times through the Greco-Roman period, are
home to
e) contains 120,000 objects from prehistoric times through the Greco-Roman period, is home to

The Egyptian Museum in Cairo, which contains 120,000 objects from prehistoric times through the
Greco-Roman period, are home to one of the most impressive collections of ancient Egyptian artifacts.

Grammar issue presented:


subject/verb agreement: subject/verb separation
which contains 120,000 objects from prehistoric times through the Greco-Roman period separates The
Egyptian Museum in Cairo (singular) from the verb are (plural)

a) which contains 120,000 objects from prehistoric times through the Greco-Roman period, are home to
Subject / verb agreement? NO (museum / are)

b) which contain 120,000 objects from prehistoric times through the Greco-Roman period, are home to
Subject / verb agreement? NO (museum / are)
Additional errors? which contain

c) containing 120,000 objects from prehistoric times through the Greco-Roman period, are home to
Subject / verb agreement? NO (museum / are)

d) which is containing 120,000 objects from prehistoric times through the Greco-Roman period, are
home to
Subject / verb agreement? NO (museum / are)

e) contains 120,000 objects from prehistoric times through the Greco-Roman period, is home to
Subject / verb agreement? YES (museum / is)
Additional error? None
(E) shows the correct subject-verb agreement; no additional errors are created.

3. A number of colorful glass vases were displayed in the store window.

a) were displayed in the store window.


b) was displaying in the store window.
c) was displayed in the store window.
d) displayed in the store window.
e) were being displayed in the store window.

A number of colorful glass vases were displayed in the store window.

Grammar issue presented:


subject/verb agreement: Quantity Words
a number always takes a plural verb

the number always takes a singular verb

a) were displayed in the store window.


Subject/verb agreement? YES (a number/ were)
Additional errors? none

b) was displaying in the store window.


Subject/verb agreement? NO (a number/ was displaying)
Additional errors? displaying

c) was displayed in the store window.


Subject/verb agreement? NO (a number/ was displayed)

d) displayed in the store window.


Subject/verb agreement? NO part of the verb ('was'/'were') is missing!

e) were being displayed in the store window.


Subject/verb agreement? YES (a number/ were)
Additional errors? being displayed is wordy

(A) is the best choice, because it shows the correct subject-verb agreement and is concise.
4. Neither of our schools students nominated for the national spelling bee were able to win the
competition.

a) Neither of our schools students nominated for the national spelling bee were
b) Neither of our schools students nominated for the national spelling bee was
c) Neither of the students from our school nominated for the national spelling bee were
d) Neither of the students nominated for the national spelling bee from our school were
e) Neither one of our schools students who was nominated for the national spelling bee was

Neither of our schools students nominated for the national spelling bee were able to win the
competition.

Grammar issue presented: subject/verb agreement: neither/either


Neither is always a singular subject and thus takes a singular verb. The original sentence uses were,
which is plural.

a) Neither of our schools students nominated for the national spelling bee were
Subject/verb agreement? NO (neither/were)

b) Neither of our schools students nominated for the national spelling bee was
Subject/verb agreement? YES (neither/was)
Additional errors? None

c) Neither of the students from our school nominated for the national spelling bee were
Subject/verb agreement? NO (neither/were)

d) Neither of the students nominated for the national spelling bee from our school were
Subject/verb agreement? NO (neither/were)

e) Neither one of our schools students who was nominated for the national spelling bee was
Subject/verb agreement? YES (neither/were)
Additional errors? This version of the sentence is wordy: neither one, who was.

(B) shows the correct subject-verb agreement and is the most concise choice.

5. Everybody at the party love the chocolate cake Shelley made.

a) Everybody at the party love the chocolate cake Shelley made.


b) Everybody at the party is loving the chocolate cake Shelley made.
c) Everybody at the party loves the chocolate cake Shelley made.
d) Everybody love the chocolate cake Shelley made at the party.
e) Everybody loves the chocolate cake Shelley made at the party.

Everybody at the party love the chocolate cake Shelley made.

Grammar issue presented: subject/verb agreement: plural /singular


Everybody is a singular subject and thus takes a singular verb
Love is the plural form

a) Everybody at the party love the chocolate cake Shelley made.


Subject/verb agreement? NO (everybody/love)

b) Everybody at the party is loving the chocolate cake Shelley made.


Subject/verb agreement? YES (everybody/is loving)
Additional errors? is loving is awkward

c) Everybody at the party loves the chocolate cake Shelley made.


Subject/verb agreement? YES (everybody/loves)
Additional errors? None

d) Everybody love the chocolate cake Shelley made at the party.


Subject/verb agreement? NO (everybody/love)

e) Everybody loves the chocolate cake Shelley made at the party.


Subject/verb agreement? YES (everybody/loves)
Additional errors? Unclear/ Change in meaning!
The phrase the cake Shelley made at the party sounds like Shelley made the cake while she was at the
party.

(C) correctly agrees subject and verb and is the most concise and clear choice.

6. The public are receiving the new mayor well, though she was mostly unheard of prior to the election.

a) The public are receiving the new mayor well, though she was
b) The public receive the new mayor well, though she was
c) The public is receiving the new mayor well, though she was
d) The public is receiving the new mayor well, though she is
e) The public are receiving the new mayor well, though she is
The public are receiving the new mayor well, though she was mostly unheard of prior to the election.

Grammar issue presented: subject/verb agreement: collective nouns


Public, although made up of members functions as a singular group and thus takes a singular verb.
Are is plural.

a) The public are receiving the new mayor well, though she was
Subject/verb agreement? NO (public/are)

b) The public receive the new mayor well, though she was
Subject/verb agreement? NO (public/receive receive is the plural form of the verb.)

c) The public is receiving the new mayor well, though she was
Subject/verb agreement? YES (public/is)
Additional errors? None

d) The public is receiving the new mayor well, though she is


Subject/verb agreement? YES (public/is)
Additional errors? she is unheard of the mayor was unheard of, but now is known to the public.

e) The public are receiving the new mayor well, though she is
Subject/verb agreement? NO (public/are)
Additional errors? she is unheard of the mayor was unheard of, but now is known to the public.

(C) shows the correct subject- verb agreement and is the most concise choice.

7. We dont yet know whom, but eventually either my brother or myself are going to take over the family
business.

a) either my brother or myself are going to take over the family business.
b) either my brother nor myself are going to take over the family business.
c) either my brother or myself will be going to take over the family business.
d) either my brother or myself taking over the family business.
e) either my brother or myself is going to take over the family business.

We dont yet know whom, but eventually either my brother or myself are going to take over the family
business.
Grammar issue presented: subject/verb agreement: or/ nor
If two subjects are joined by or or nor, the verb should agree with the subject that is closer to it. In this
case, the verb are going is plural, and the subject myself is singular.

a) either my brother or myself are going to take over the family business.
Subject / verb agreement? NO (myself/ are)

b) either my brother nor myself are going to take over the family business.
Subject / verb agreement? NO uses eithernor. This construction does not exist! It is always either
or OR neithernor. Agreement is therefore impossible.

c) either my brother or myself will be going to take over the family business.
Subject / verb agreement? YES (myself will be)
Additional errors? will be going is extremely awkward.

d) either my brother or myself taking over the family business.


Subject / verb agreement? NO - missing verb - myself/ (is) taking

e) either my brother or myself is going to take over the family business.


Subject / verb agreement? YES (myself / is)
Additional errors? none

(E) correctly agrees subject with verb and remains a clear and well-written sentence.

8. Next to me on the bench sits two older women.

a) on the bench sits two older women.


b) on the bench sit two older women.
c) two older women sitting on the bench.
d) sit on the bench two older women.
e) two older women sits on the bench.

Next to me on the bench sits two older women.

Grammar issue presented: subject/verb agreement: subject / verb / object


In this sentence the verb: sits precedes the subject: two older women
the bench is the object of the sentence.
a) on the bench sits two older women.
Subject / verb agreement? NO women (plural) / sits (singular)

b) on the bench sit two older women.


Subject / verb agreement? YES women (plural) / sit (plural)
Additional errors? None

c) two older women sitting on the bench.


Subject / verb agreement? NO missing verb - women / (are) sitting

d) sit on the bench two older women.


Subject / verb agreement? YES women (plural) / sit (plural)
Additional errors? Awkward construction: Next to me sit on the bench two older women.

e) two older women sits on the bench


Subject / verb agreement? NO women (plural) / sits (singular)

(B) correctly agrees subject and verb and is well constructed.

2. Modifiers: Introduction

Modifiers

A. Introduction
B. Adjectives and Adverbs
C. Adjectives and Adverbs with Sense Verbs
D. Misplaced Modifiers
E. Sample Questions

Modifiers are words, phrases, or clauses that provide descriptive detail about other words,
phrases, or clauses. Adjectives (the red car, the happy child) are modifiers, as are adverbs (he runs
quickly).

Adjectives modify nouns.

Adverbs modify verbs or adjectives.


Sometimes, however, entire phrases can be used as modifiers. Modifying phrases function the same
way as single-word modifiers do, but because they're often buried in an already complicated sentence,
they can be harder to spot than adjectives and adverbs. Lengthy modifiers therefore appear quite often
on the GMAT. This chapter will give you more detailed tips and methods by which to recognize these
kinds of modifiers on the GMAT. But for general reference, keep this rule in mind: Any part of a
sentence that adds extra information can be considered a modifier.

Our list of common modifier errors, begins with adjectives and adverbs, and then considers
phrases and clauses.

2-B. Modifiers: Adjectives and Adverbs

Errors in the Use of Adjectives and Adverbs.

The first step in identifying modifiers is to read the sentence and look for descriptive single words. Once
you have done this, you should then look at each and try to determine whether it is an adjective or an
adverb.

1. An adjective describes a noun or pronoun and answers the questions: how many, which one,
what kind?

She is a good tennis player. (What kind of tennis player is she?)

This is an easy exercise. (What kind of exercise is it?)

2. An adverb describes a verb, an adjective or another adberb, and answers the questions:
when, where, how, why, and to what extent?

She plays tennis well. (She plays how?)

This exercise is relatively easy. (To what extent is it easy?)

An easy way to identify adverbs, or to distinguish them from adjectives, is to look at the ending. Most
adverbs are formed by adding -ly to the adjective, such as: He worked quickly.
However, there are a few exceptions that you should memorize, if you're not already familiar with them.
Here's a list of common exceptions to study.

Exceptions
The following irregular adverbs do not end in ly.
Adjective Adverb
early early (ends in -ly, but so does the adjective)
fast; faster; fast; faster; fastest
fastest
slow slow (meaning at an incorrectly slow speed)
good well, ill (meaning badly, as in "to think ill of")
better; best better; best
hard hard (hardly means almost not)
late late (lately means recently)
worse; worst worse; worst
little little (meaning not much, or not at all)
more; most more; most
less; least less; least
much much
very very
far; farther; far; farther; farthest
farthest
further; furthest further; furthest
close; closer; close; closer; closest (closely means in great detail)
closest
near; nearer; near; nearer; nearest (nearly means almost)
nearest
high; higher; high; higher; highest (highly means very, or very well,
highest as in "to think highly of")
low; lower; lowest low; lower; lowest (lowly means humble, adj., or in a
low position, adv.
wide; wider; wide; wider; widest (widely means generally)
widest
long; longer; long; longer; longest
longest
short; shorter; short; shorter; shortest (several meanings; shortly
shortest means soon)
deep; deeper; deep; deeper; deepest (deeply means very)
deepest
ago ago

More Exceptions
The following irregular adverbs do not end in ly.
either (meaning also)

pretty (meaning moderately)

quite

rather

almost

tall (meaning to a given standard, as in "to stand tall")

After you've identified the word as an adjective or adverb, the next step is to determine whether
it is used correctly. Look at the sentence below:

She is a real good swimmer.

This sentence contains a descriptive word modifying a noun, and another descriptive word modifying an
adjective. Are these modifying words used correctly? It's hard to tell, because they're all grouped
together. Break the sentence into parts:

As you can see, the word good modifiers swimmer. Good is an adjective, and swimmer is a noun.
Adjectives modify nouns, so no error there. But notice the word real, used to modify the adjective good.
Real is an adjective and only adverbs modify adjectives.

In this version, the adjective real, which modifies the adjective good, is replaced with an adverb really.
Note the difference: really is real with an ly tacked on.

Incorrect: She is a real good swimmer.


Correct: She is a really good swimmer.

Incorrect: The new student speaks poor.

Correct: The new student speaks poorly.

This sentence contains one descriptive word modifying a noun, and one descriptive word modifying a
verb. In both versions, the adjective "new" is used to modify the noun "student," which is correct. In the
first version, however, the word "poor" is used to modify the verb "speaks". But "poor" is an adjective -
and adjectives cannot modify verbs. Therefore, the second version correctly replaces the adjective
"poor" with the adverb "poorly". Once again, the difference between the two is a mere, but necessary, "-
ly".

2-C. Modifiers: Adjectives and Adverbs

Errors of Adjectives and Adverbs with Sense Verbs.

The following verbs require adjective modifiers:

sound look smell taste feel seem

These verbs are all "sense verbs," or verbs that describe someone's sensation or feeling or
perception. Unlike other verbs, they require adjective, not adverb, modifiers.

Incorrect: The strawberry shortcake tastes deliciously.

Correct: The strawberry shortcake tastes delicious.

Sense verbs convey personal opinions, thoughts, and perceptions in an inherently subjective manner
that is, they describe someone's personal experience. The sentence The strawberry shortcake tastes
delicious has essentially the same meaning as "The strawberry shortcake tastes delicious to me" or "I
think the strawberry shortcake tastes delicious." Because each sentence describes the attributes of the
shortcake as seen through the eyes (and mouth) of an observer, each sentence should use the same
version of the modifier: the delicious shortcake. Another way to approach this sentence is to think about
it as a sandwich. When a sense verb is sandwiched between a noun and a modifier, the modifier
should always agree with the noun.

Some sense verb modifiers are commonly misused in speech. Be especially careful with these: just
because they sound right doesn't mean they are right. Sometimes these errors arise from the
misinterpretation of a popular grammar rule. Here's a common example:

After she returned from the three-week vacation, she looked very well.

How many times have you heard someone say, "He looks well"? It probably sounds fine, but in fact, this
sentence is a comment on the visual abilities of the man in question; it means something like, "He's
skilled at looking." Pretty funny, right? But why is it wrong?
Looking at the version above: if you place an adverb (well) directly after a verb (looked), then the
adverb modifies the verb. But we don't want to describe a verb we want to describe a noun (or
pronoun), in this case a woman who just came back from vacation.

"She" is a pronoun, and pronouns (which stand in for nouns) are modified with adjectives. Thus the
correct sentence fixes our modification problem by replacing the adverb "well" with the adjective "good".

Incorrect: After she returned from the three-week vacation, she looked very well.

Correct: After she returned from the three-week vacation, she looked very good.

2-D. Modifiers: Misplaced Modifiers

Location of Modification Misplaced Modifiers

What's wrong with this sentence?

Finally thinking clearly, the book was able to be understood by Rebecca.

The meaning of the sentence seems clear enough: that Rebecca finally understood the book after she
started thinking clearly.

But what does the sentence actually say? If you look closely, you'll see that, because of the placement
of certain words, the sentence makes the book, not Rebecca, the subject of the sentence: which makes
it seem as if the book were thinking clearly, not Rebecca. That's funny how can a book think clearly?
but not what we meant at all. What went wrong?

If you'll recall, modifiers are often adjectives or adverbs. But modifiers can also be groups of words
known as adjectival or adverbial phrases or clauses that describe another part of the sentence.
Like single-word adjectives and adverbs, these multiple-word modifiers must be placed as close
as possible to the word or group of words they're modifying. Those modifiers that fail to observe
this rule are called "misplaced modifiers".
Misplaced modifiers can be highly deceptive - and are extremely common on the GMAT. Because we
know what the sentence means to say, it's easy to miss this type of placement error, unless we have our
eyes open for them.

Let's look again at the example:

Even though the modifier is followed immediately by the book, we might very easily assume that
because a book can't think, we can overlook its placement in the sentence, as the phrase Finally
thinking clearly must refer to Rebecca. But the GMAT isn't testing our ability to understand mangled
sentences; it's testing our understanding of English grammar. And according to the rules of English
grammar, a modifier must always be placed as close as possible to the word it's modifying. Thus,
the modifier in this sentence must be describing the book. So this sentence is misleading - and
incorrect - because the modifier Finally thinking clearly is not immediately followed by what it is
modifying: that is, Rebecca.

Try this next example:

On arriving at the train station, his friends greeted Jay and took him immediately to his speaking
engagement in Springfield.

Once again, it's likely that this sentence sounds fine at first glance: Jay gets to the train station, after
which his friends meet him and take him to his important engagement. Take a closer look: let's break it
down, and check to make sure that the modifiers (and the objects being modified) are placed where
they belong.

First find the modifying phrase: look for a descriptive group of words set off by a comma or
commas. Here, we have one phrase that looks like that: On arriving at the train station. After identifying
the modifier, the next step is to figure out which word/s it should be modifying, and which word/s it is
modifying. Who is arriving at the train station - Jay or his friends? Because the modifier is followed
directly by the phrase his friends, it sounds like Jay's friends are arriving, rather than Jay himself. We
want Jay to be arriving at the station.

In this version of the sentence, it's suddenly quite clear that Jay arrived at the train station, and his
friends greeted him and took him away. This was accomplished by placing the modifier On arriving at
the train station right next to Jay - the person whom the modifier was meant to describe.

Incorrect: On arriving at the train station, his friends greeted Jay and took him immediately to

his speaking engagement in Springfield.


Correct: On arriving at the train station, Jay was greeted by his friends, who immediately took

him to his speaking engagement in Springfield.

Misplaced modifiers won't always occur at the beginning of sentences: any descriptive phrase or clause
is a potential misplaced modifier. Just make sure the modifying phrase or clause is as close as
possible to the word/s being modified.

Descriptive phrases are not always set off by commas. These pronouns often indicate modifying
phrases:

which (refers to things)

that

who (refers to people)

whose

whom

In addition to helping you identify modifying phrases, these pronouns can be helpful when you're trying
to fix a seemingly incorrect sentence. Look at the examples below:

Sounds Funny: Joan's father, preferring meat to vegetables, made a breakfast of eggs and

bacon every morning.

Better: Joan's father, who preferred meat to vegetables, made a breakfast of eggs and bacon

every morning.

Sounds Funny: Your tea kettle, having a leak in the bottom, was thrown away last week.

Better: Your tea kettle, which had a leak in the bottom, was thrown away last week.

Note the different uses of "who" and "which": "who" is used in the first example because it introduces a
phrase that describes a person ("Joan's father"). "Which" is used to introduce a phrase that describes a
thing (the "tea kettle"). "That" is also used to describe things, as opposed to people.

Don't forget!
Its is the possessive of it, and it's is the contraction of it and is.

The dog licked its paw.


It's about to rain.

2-E. Modifiers: Sample Questions

EASY

1. Previously thought to have been extinct, a team of biologists rediscovered the New Caledonia crested
gecko in 1994.

a) a team of biologists rediscovered the New Caledonia crested gecko in 1994.


b) a team of biologists, in 1994, rediscovered the New Caledonia crested gecko.
c) in 1994 the New Caledonia crested gecko was rediscovered by a team of biologists.
d) in 1994 a team of biologists rediscovered the New Caledonia crested gecko.
e) the New Caledonia crested gecko was rediscovered by a team of biologists in 1994.

Explanation: This question tests modifiers. The modifier Previously thought to have been extinct refers
to the New Caledonia crested gecko. Thus the modifier must immediately precede that which it is
modifying. Only (E) does this. (A) and (B) change the meaning of the sentence to make the biologists
appear to have been extinct, while in (C) and (D) the modifier is not modifying anything at all.

HARD

2. Erasmus's tomb lies inside the Basel Munster, located in Switzerland, an architectural monument
which having survived medieval earthquakes, and remains one of Switzerland's most well-known
buildings to this day.

a) Erasmus's tomb lies inside the Basel Munster, located in Switzerland, an architectural monument
which having survived medieval earthquakes, and
b) Erasmus's tomb lies inside Switzerland's Basel Munster, an architectural monument which survived
medieval earthquakes and
c) Switzerland's Basel Munster, a monument of architecture which survived medieval earthquakes,
houses Erasmus's tomb
d) The Basel Munster, in Switzerland, an architectural monument which, having survived medieval
earthquakes, is now home to the tomb of Erasmus
e) The tomb of Erasmus, being housed inside Switzerland's Basel Munster, is an architectural
monument which survived medieval earthquakes and

Explanation: This question tests misplaced modifiers, as well as changes in meaning and wordiness.
(A) places the modifier (an architectural monument) directly after Switzerland, which, though not as
confusing as some misplaced modifiers, is still incorrect, as other choices are offered which place the
modifier closer to the intended subject, Basel Munster. (C), while free of grammatical errors, changes
the meaning of the sentence by making the Basel Munster the subject of the sentence instead of
Erasmus's tomb. (D) does the same, and is also excessively wordy and omits the necessary and at the
end of the underlined portion. (E) is excessively wordy, and includes the word being, which is awkward
in this context. (B), the best choice, is free of grammatical and stylistic errors, and maintains the
meaning of the original sentence.

3. Parallelism: Introduction

Parallelism

A. Introduction
B. Lists of Verbs and Parallel Constructions
C. Lists of Adjectives or Adverbs
D. Comparisons Between Multiple Pronouns
E. Sample Questions

"Parallelism" refers to sentences in which all items are described in the same format. Unlike
some of the other grammatical topics covered in this chapter, parallelism is a pretty intuitive concept to
master; there are no exceptions to memorize, no strange rules to remember. Once you understand the
concept, you're pretty much good to go. But why, if it's so simple, is parallelism included so often on the
GMAT? For the same reason that misplaced modifiers, subject-verb agreement, and other "simple"
topics are included: because parallelism can be tricky to recognize.

How to recognize a parallelism


Parallelism is a rule of English grammar that demands consistency in a
sentence's structure. Any lists of ideas, places, activities, or descriptions that
have the same level of importance whether they be words, phrases, or clauses -
must be written in the same grammatical form. Some examples:

activities: running, biking, and hiking

places: the store, the museum, and the restaurant

ideas: how to read, how to write, and how to learn

descriptions: quickly, quietly, and happily

Note the grammatical consistency in each list: the activities all end in ing; try to
be consistent with articles such as the, and or a; the ideas all begin with how to; the
descriptions are all adverbs . In each list, whatever grammatical form is applied
to one item is applied to all items. On the GMAT, this rule what applies to one
must apply to all is pretty much all you need to remember.

Parallelism means something very similar to what it means in mathematics. Think of parallel lines:
They're straight, they're equally spaced, and they're very clearly "parallel".

To translate this mathematical concept to grammar, first think of a sentence. A sentence can be split up
in many different ways: by word, by phrase, by part of speech, by items in a list. What parallelism says is
that these similar parts of a sentence must "track" one another, in the same way that parallel lines track
one another. For example, every item in a list must use the same form as the others.

Think of it like this: pretend that the parts of a sentence are lined up, one on top of the next, along their
own parallel lines. Consider the sentence "Joe was trying to decide between eating, running, and to
walk to the store". There are three items in the list of activities that Joe is considering, so separate these
and imagine them on their own parallel lines:

eating

running

to walk

To be parallel, all verbs must look identical. In this case, one sticks out like a sore thumb: "to walk".
Here's the correct version:

eating

running

walking

The concept of parallelism is easy to master - but recognizing a parallelism question is more
difficult. This section will show you how to do both.

3-B. Parallelism: Lists of Verbs and Parallel Constructions

All elements in a list whether it's a list of nouns, of infinitives, of gerunds, of prepositional
phrases, or of clauses - should be in similar form. "Similar form" means that all of the items in
the list must agree.

On the test, you'll often see a list of three verbs, in which two agree, but one does not. In order for the
sentence to be correct, all three verbs must agree:

Patty ate macaroons, drank soda and was dancing the tango.

This is a list of activities more specifically, those activities undertaken by Patty. Parallelism dictates
that all the things Patty did must be listed in the same form, and since "all the things Patty did" are
verbs, all verbs in the sentence must agree in tense and number. Do they?
This chart identifies each verb form in the sentence. The list of verbs in this version of the sentence
contains two singular simple past tense verbs (ate and drank) and one singular past progressive verb
(was dancing). Because the verbs are placed together in a list, this cannot be correct. The verbs should
all match:

This version correctly changes the mismatched past progressive verb, was dancing, to the simple past
tense, danced, so that it looks and sounds exactly like the other verbs in the list, ate and drank. This
sentence now exhibits proper parallelism.

Incorrect: Patty ate macaroons, drank soda and was dancing the tango.

Correct: Patty ate macaroons, drank soda and danced the tango.

Here's another example using a list of gerunds:

Incorrect: All business students should learn word processing, accounting, and how to

program computers.

Correct: All business students should learn word processing, accounting, and computer

programming.

The verb "to program" must be changed to "programming," because the rest of the verbs are already in
the -ing form.

You'll often see lists of infinitives on the GMAT: the "to ___" verbs (to walk, to talk, to eat, to chat, to
drink). With infinitives, a very simple rule applies: the word "to" must either go only before the first verb
in the list, or before every verb in the list. For example:

Correct: He likes to swim, to sail, and to dance.

Correct: He likes to swim, sail, and dance.


Incorrect: He likes to swim, sail, and to dance.

The first two sentences are equally acceptable variations. The third sentence is incorrect because it
lacks consistency; the verb changes from to swim to sail, and then back to to dance. This violates the
rules we've laid out.

List of infinitives: Options

To ______________, ______________, and ______________.

To ______________, to ______________, and to ______________.

The principle governing lists of infinitives applies, in fact, to any words that might
come before each item in a series: prepositions (in, on, by, with), articles (the, a,
an), helping verbs (had, has, would) and possessives (his, her, our). Either repeat
the word before every element in a series or include it only before the first item.
Anything else violates the rules of parallelism.

Here is a full list of possible parallel constructions, and examples of each:

Nouns Nouns with Adjective


clocks Adjectives Modifiers
books clumsy red balloon
sofas rusty bicycle
cheerful

bright

Verbs Participial Phrases Infinitives


walked mastering a language play guitar
talked reading a novel write a book
chewed gum make a discovery
or
Adverbial Phrases to play guitar
Adverbs in frustration
to write a book
bitterly in anger
to make a discovery

sadly

3-C. Parallelism: Lists of Verbs and Parallel Constructions


Just like how verbs, adverbs or adjectives in a list must agree. Descriptive words are easy to
replace with wordy phrases, and test writers will try to trip you up by including a verb or phrase
among a list of adjectives or adverbs:

On the morning of his fourth birthday, Johnny was giggly, energetic, and couldn't wait for the party to
begin.

If you read through the sentence quickly, it might sound acceptable. However, the list includes one item
that doesn't belong:

This looks to be a list of adjectives until you reach the third item in the list: it's not an adjective, it's a
verb! The "list of adjectives" won't be complete until the last item falls into step with the others:

This example replaces the verb phrase couldn't wait with the descriptive phrase very eager which
indeed includes an adjective.

Watch for consistency in item type as well as consistency of form.

Incorrect: On the morning of his fourth birthday, Johnny was giggly, energetic and couldn't wait

for the party to begin.

Correct: On the morning of his fourth birthday, Johnny was giggly, energetic and very eager for

the party to begin.

3-D. Parallelism: Lists of Verbs and Parallel Constructions

Sometimes, you'll come across sentences with multiple pronouns. In many cases, parallelism
requires that the pronouns be identical.
Incorrect: Those who exercise in addition to maintaining a healthy diet are likely to be in better

health than the people who maintain a healthy diet but don't exercise.

Correct: Those who exercise in addition to maintaining a healthy diet are likely to be in better

health than those who maintain a healthy diet but don't exercise.

In the first sentence, the pronoun those who, in the first part of the sentence, is matched with the phrase
"the people who" in the second part of the sentence. Notice how much cleaner and easier to understand
the second sentence is.

Look at the sentence below:

If one decides to break the law, they must be willing to take responsibility for any repercussions.

This sentence contains two pronouns. Do they match?

When using the word one as a pronoun referring to an unspecified person, the only acceptable match is
one. The first sentence inserts they instead, which is incorrect. The same rule applies for the pronoun
you when it's used to refer to an unspecified person. The GMAT does not prefer one to the other, but
one and you cannot be used interchangeably in the same sentence:

Incorrect: If one decides to break the law, you must be willing to take responsibility for any

repercussions.

Correct: If one decides to break the law, one must be willing to take responsibility for any

repercussions.

Correct: If you decide to break the law, you must be willing to take responsibility for any

repercussions.

Both of latter versions are correct.

Be consistent: use whichever pronoun you choose all the way through.
Final tips on recognizing a parallelism
Look for:

Lists

Clauses or phrases separated by commas

Comparisons using multiple pronouns

3-E. Parallelism: Sample Questions

EASY

1. Some of the many renovations set for Memorial Field in the coming years include building additional
seating, improving safety, and the construction of a new varsity athletics center.

(A) and the construction of a new varsity athletics center.


(B) and constructing a new varsity athletics center.
(C) and also the construction of a new varsity athletics center
(D) and a new varsity athletics center
(E) and a new varsity athletics center under construction.

This question tests parallelism. All items in a list must be parallel, meaning they must be in the same
format grammatically. The original list in (A) is not parallel. the construction does not match building
additional seating and improving safety. The last item must be changed to constructing in order to fit the
progressive verbs building and improving. Only (B) does this.

HARD

2. Richard is not only a terrific pianist, but also great at playing hockey.

a) Richard is not only a terrific pianist, but also great at playing hockey.
b) Richard not only is a terrific pianist, but also is great at playing hockey.
c) Not only great at playing hockey, Richard also is a terrific pianist.
d) Richard is not only a terrific pianist, but also a great hockey player.
e) Also great at playing hockey, Richard is a terrific pianist.

This question tests parallelism. Not onlybut also is a case for parallelism, meaning that both things
which are compared must be in the same grammatical format. A terrific pianist does not match great at
playing hockey because pianist is a noun and playing hockey is a verb/noun. What does match a terrific
pianist is a great hockey player. Only (D) does this and is therefore parallel.

3. The philosophical doctrine of Incompatibility posits an inherent irreconcilability among the doctrine of
Determinism, in philosophy, which holds that each state of affairs is necessitated by the states of affairs
that preceded it and the existence of free will.

A. among the doctrine of Determinism, in philosophy, which holds that each state of affairs is
necessitated by the states of affairs that preceded it and the existence of free will
B. between the doctrine of Determinism, holding each state of affairs as necessitated by the states of
affairs that preceded it, and the existence of free will
C. in the doctrine of Determinism, which in philosophy holds the idea that each state of affairs is
necessitated by the states of affairs preceding, and free will's existence
D. between the doctrine of Determinism, which holds that each state of affairs is necessitated by the
states of affairs preceding it, and the existence of free will
E. among the doctrine of Determinism, which holds that each state of affairs may be necessitated by the
states of affairs preceding it, and free will existing

This question tests redundancy, parallelism, and idiom usage. Because the non-underlined portion of
the sentence establishes the context within philosophy, choices A and C, which repeat in philosophy, are
redundant and wordy. By introducing what is supposed to be the description of Determinism with the
word holding 'instead of which holds, choice B implies that Incompatibility holds the idea, not
Determinism. Choice E fails to maintain parallelism (which holds and existing). Choices A and E, and C,
also use incorrect word choice: because the irreconcilability involves only two things, the word 'among'
(used in A and E), which refers to more than two, is incorrect; irreconcilability in' (choice C) is incorrect
as well; it should be irreconcilability between. Choice D contains no errors in grammar or usage, and is
the best choice. The answer is D.

4. Pronoun Agreement: Introduction

Pronoun Agreement

A. Introduction
B. Subject vs. Object
C. Who vs. Whom
D. Singular and Plural Pronouns
E. Possessive Pronouns
F. Objects of to be verbs
G. Relative Pronouns
H. Sample Questions

Pronouns stand in for nouns in a sentence. When replacing any noun (Matt, the cheerleader, the
chair) with a pronoun (he, she, it), the pronoun must match the form of the missing noun. A noun has
three elements: number, gender, and case.
Number: A noun may be either singular (one) or plural (more than one) and the pronoun must

reflect that. For example: Matt went to the store. Because Matt is singular (one person) the correct
pronoun would be he as in He went to the store and not They went to the store. The pronoun they would
be appropriate only if more than one person were going to the store. For example, The students went to
the store would change to They went to the store because there is more than one person.

Gender: This refers to whether the noun is masculine or feminine. The pronoun must reflect this.

For example, Matt went to the store should be rewritten as He went to the store and not She went to the
store because Matt is male.

Case: Keep in mind whether the noun is the subject or object of the sentence and use the

appropriate pronoun. The subject of the sentence is the noun that is doing the action. In the sentence
Matt went to the store, Matt is the person going, so replace Matt with he: He went to the store. The
subject of the sentence is the thing receiving the action. Take the sentence The ball struck Matt in the
arm. In this case Matt is the one being struck, therefore use the objective pronoun (him). The ball struck
him in the arm.

Because pronouns follow the same agreement rules as nouns, it is important to be clear about what
noun it is replacing. The first step in tackling a pronoun question is to locate and identify the
pronouns in the sentence.

Simple Pronouns Possessives


Used as Used as
Subject Object
Adjective Pronoun
I me my mine
you you your yours
he him his his
she her her hers
it it its its
we us our ours
they them their theirs
everyone everyone everyone's everyone's

4-B. Pronoun Agreement: Subject vs. Object

Once you've found a pronoun in a Sentence Correction question, check whether it's acting as
the SUBJECT or the OBJECT of the sentence or phrase. Is the following sentence correct or
incorrect?

How could she blame you and he for the accident?


The first step is to identify the pronoun(s). There are three in this sentence: "she," "you," and "he":

Next, try to define whether each pronoun is acting as a subject or object. Here, she is the subject, and
the pronouns you and he are acting as the objects of the sentence:

How do we know this? Because she is doing the action (blaming) and you and he are receiving it
(getting blamed). However, he does not seem to be in the correct form. Refer to the chart in the previous
section, or to the proper answer to the question "Who did she blame?", which is him not he. ("Who did
she blame? She blamed him.")

Both pronouns acting as objects must be in the objective case; as indicated in the graphic above, him is
objective while he, used in the first sentence, is subjective, and therefore incorrect

Incorrect: How could she blame you and he for the accident?

Correct: How could she blame you and him for the accident?

Let's look at another example:

Incorrect: Her was better suited for the job.

Correct: She was better suited for the job.

Here, the pronoun is the subject of the sentence, as the job is clearly not the subject, and there are no
other nouns in the sentence. Because the pronoun stands in for "the woman" (some woman), the
pronoun should be the subject form of the her/she pronoun as indicated by the chart: meaning, "she".

Now let's look at a case that often causes confusion:


John and me drank a bottle of wine.

Because it's confused so often in spoken language, it can be difficult to tell when the pronoun in the
phrase "someone else and me/I" is used incorrectly. But it's actually quite easy to remember when to
use "me", and when to use "I": cross out everything in the "someone else and me/I" phrase except the
pronoun.

"Me drank a bottle of wine" sounds like caveman-speak and the proper pronoun is clearly "I".

Incorrect: John and me drank a bottle of wine.

Correct: John and I drank a bottle of wine.

Let's try it again on the following sentence:

The dinner was eaten by John and I.

Perform the test:

The dinner was eaten by John and I. ?


or
The dinner was eaten by John and me. ?

The second sentence is grammatically correct ("I/me" is acting as the object), so the proper pronoun is
"me." This test works for many instances of misused pronouns, but to be safe, you should memorize the
subject/object pronoun chart.

Incorrect: The dinner was eaten by John and I .

Correct: The dinner was eaten by John and me.

4-C. Pronoun Agreement: Who vs. Whom

'If the pronoun is acting as a subject, use who. If it is acting as an object, use whom.

I don't know whom Kate married.


Why is "whom" correct? Because Kate is the subject of this sentence not the person she married. To
simplify who/whom questions, try rearranging the sentence into a question, and then answer it. Let's try
it:

Question: Who/m did Kate marry?


Answer: Kate married him.

You wouldn't say "Kate married he," right? Since the pronoun used in the answer is "him," an object
pronoun, the pronoun in the original sentence should also be an object pronoun: whom.

Here's another one to try:

Who took out the trash?

Because the sentence is already a question, you can't run the test as we did above. But not to worry: all
you need to do is answer the question:

He took out the trash.

You wouldn't say "him took out the trash," so the pronoun in the original sentence must match the form
of the pronoun "he," which is a subject pronoun: who is correct.

4-D. Pronoun Agreement: Singular and Plural Pronouns

Pronouns also act like nouns in the realm of verb agreement. For some pronoun questions, you
also need to check if the pronoun and its verb agree in number.

All of the following pronouns are singular:

anyone anything each

either everyone everything

neither no one nothing

what whatever whoever

These are plural:


both several few

many others

Consider these two sentences:

Incorrect: Everyone on the project have to come to the meeting.

Correct: Everyone on the project has to come to the meeting.

Referring to the chart above, you'll see that the pronoun everyone is singular. Its verb must therefore be
singular as well: has is correct, not have.

Incorrect: Neither his bodyguards nor he was were there. (was is correct!)

Correct: Neither he nor his bodyguards were there.

As covered in an earlier section of this chapter, the constructions "either... or" and "neither.. .nor" always
take the verb form that matches the noun that is closer to the verb. (In these constructions, either and
neither are actually not pronouns at all, but conjunctions.)Thus, were is incorrect in the first sentence
because he a singular pronoun, is closer to the verb than bodyguards a plural noun; but were is correct
in the second sentence because the order of the subjects is reversed, so that the plural noun
bodyguards is closer to the verb.

4-E. Pronoun Agreement: Possessive Pronouns

When you come across possessive pronouns such as yours, theirs, his, and hers, check to see whether
they agree with other pronouns in the sentence. Most possessive pronouns are used messily in
spoken language, so be careful to take special note when you see two pronouns in a sentence.

Incorrect: I brought my beer, and I'm glad to see that some of you brought theirs.

Correct: I brought my beer, and I'm glad to see that some of you brought yours.

In this sentence, the possessive pronoun towards the end of the sentence should match the pronoun
following "Some of". Because the earlier pronoun is "you", the possessive pronoun must be yours, not
theirs. Theirs would only be correct if the clause began ...and I'm glad to see that some of them
brought...

Incorrect: If anyone comes over to take your name, you take theirs.
Correct: If anyone comes over to take your name, you take his or hers.

The subject is anyone, which is singular, and which therefore requires a singular pronoun such as his or
hers. This error has become common because of the demand for political correctness; instead of saying
his or hers, people will often just say theirs. Either his or hers alone is technically correct, but writing his
or hers, is also acceptable.

4-F. Pronoun Agreement: Objects of to be verbs

Very simple: objects of to be verbs are in the subject form. Watch for pronouns following "to be"
verbs such "it should have been," "it is," "it could have been," "it was", and make sure they are in subject
form. This is another error common in speech; but it's easy to identify.

Incorrect: It must have been her who called.

Correct: It must have been she who called.

"It must have been" is a "to be" verb, so the pronoun must be in subject form: she, not her.

4-G. Pronoun Agreement: Relative Pronouns

Which, that and who are relative pronouns. A relative pronoun must refer to the word
immediately preceding it. If the meaning of the sentence is unclear, the pronoun is in the wrong
position.

Which introduces non-essential clauses; that introduces essential clauses

Who refers to individuals; that refers to a group of persons, class, type, species, or one or more

things.

Incorrect: John was met at the door by a strange man, which he, being afraid, opened slowly.

Did John open the man? Probably not. This sentence is definitely confusing, but its meaning can be
clarified by adjusting the placement of the nouns in the sentence.

Correct: John was met by a strange man at the door, which he opened slowly out of fear.

It's now clear what John is opening, and why.

On the GMAT, the pronouns "one" and "you," which are part of a class of pronouns called "impersonal
pronouns," are often improperly matched with their respective possessive pronouns. You may have
heard that using "you" is less proper than using "one," but on the GMAT, all that matters is that
the pronouns agree there's no word-choice preference one way or the other. Look at these
examples:
Incorrect: One should have their teeth checked every six months.

Correct: One should have one's teeth checked every six months.

Correct: One should have his or her teeth checked every six months.

Correct: You should have your teeth checked every six months.

Incorrect: One should take your responsibilities seriously.

Correct: One should take one's responsibilities seriously.

Correct: One should take his or her responsibilities seriously.

Correct: You should take your responsibilities seriously.

As long as one isn't paired with your, or you with one's, the sentence is probably correct.

A summary of how to recognize pronoun errors.


Look for:
Subject or object pronouns
Who or whom
Pronoun agreement
Relative pronouns

4-H. Pronoun Agreement: Relative Pronouns

EASY

1. The choir sang passionately, as they moved through elaborate and challenging four-part harmonies.

a) as they moved through elaborate and challenging four-part harmonies.


b) as they were moving through elaborate and challenging four-part harmonies.
c) moving themselves through elaborate and challenging four-part harmonies.
d) as it moved through elaborate and challenging four-part harmonies.
e) moving elaborately through challenging four-part harmonies.
Explanation: This question tests pronoun agreement. The sentence in (A) contains incorrect agreement
because The choir is singular. Though it is made up of many members, the choir is a group and
functions as a singular subject. Thus, the pronoun referring to the choir should also be singular. They is
plural. (B) uses they and (C) uses themselves Thus neither is correct. (E) changes the meaning of the
sentence, by rewording it such that the movement is elaborate, not the harmonies. Only (D) uses the
pronoun it to agree with the singular subject the choir.

HARD

2. Marston was an early seventeenth century dramatist and it is likely that him and Shakespeare
borrowed ideas from one another.

a) it is likely that him and Shakespeare borrowed ideas from one another.
b) it is likely that they borrowed ideas from one another.
c) it is likely that him and Shakespeare borrowed ideas from each other.
d) it is likely that himself and Shakespeare borrowed ideas from one another.
e) it is likely that he and Shakespeare borrowed ideas from one another.

Explanation: This question tests pronoun agreement. The pronoun him in the original sentence
replaces Marston. It is in the wrong case. Instead of the objective case, the pronoun should be in the
subjective case, since Marston is the subject of the sentence. Therefore (E), which uses the subjective
pronoun he, is correct. (B) is wrong because, though they is subjective, it eliminates any meaning of
Shakespeare from the sentence, making the pronoun ambiguous.

5. Verb Time Sequences: Introduction

Verb Time Sequences

A. Introduction
B. Verb Tense
C. Mood
D. Voice
E. Sample Questions

Mastering verb usage is extremely important in conquering the verbal portion of the GMAT. Here is a
detailed primer on tense:

A. Time

"Tense" tells about time. That is, a verb's tense indicates when the action specified by the verb took
place. An action or event can take place in the past, the present, or the future.

1. Present

Verbs in the present tense indicate an action that is taking place right now, in the present moment
only. Present tense verbs can also indicate unchanging states of being or action, or repeated actions:

I am happy right now.


Here, the present tense verb am indicates something happening right now: the speaker is happy at this
present moment in time.

I am a happy person.

In this sentence, the present tense verb am indicates an unchanging state: the speaker is generally a
happy person, or that his state of being is one of unchanging happiness.

I study Greek for two hours every day.

Here, the present tense verb study indicates a repeated action: the speaker studies once a day, every
day.

Present tense verbs are formed by taking the uninflected (unaltered) form of the verb, which is found by
removing "to" from the infinitive form: for example, the present tense walk from the infinitive to walk, the
present tense talk from the infinitive to talk, or the present tense eat from the infinitive to eat.

2. Past

Verbs in the past tense indicate an action that took place in the past: that is, at some point prior to the
present moment.

Yesterday, I walked to the store.

Here, the past tense verb walked indicates an action that took place yesterday, and is no longer
happening.

Past tense verbs are usually formed by adding an ed to the uninflected (or present tense) form of the
verb: for example, talk becomes talked, walk becomes walked and balk becomes balked. (Irregular
verbs, such as to eat and to have, are not formed in the same way; if you are unfamiliar with these
forms, consult a basic English grammar guide.)

3. Future

Verbs in the future tense indicate an action that takes place in the future: that is, at some point after the
present moment.

Tomorrow, I will eat dinner with friends.

Here, the future tense verb will eat indicates an action that will occur, in its completion, tomorrow.

Future tense verbs are usually formed by adding will or shall to the uninflected (or present tense) form of
the verb: for example, talk becomes will talk or shall talk, walk becomes will walk or shall walk, eat
becomes will eat or shall eat.

B. Aspect

In addition to an event's place in time, verb forms can also indicate how long the event continued, or for
what duration it occurred. "Aspect" describes the event's completion, duration, or repetition: did the
event happen only once, in the past? or did it begin to happen in the past, and still happening? or will it
happen, and continue to happen for some time, at some later time? or does it happen every so often,
but not continuously?
Each basic tense (past, present, future) has a perfect, progressive, and perfect progressive forms:

1. The perfect form indicates an action that is completed.

Past Perfect is used to describe events or activities that took place in the past, before another event or
action. The past perfect is formed by adding had to the past participle form of the verb, which usually
ends in -ed:

By the time Michael arrived, the party had ended.

Present Perfect is used to describe events or activities that were begun in the past. The present perfect
is formed by adding was or were to the past participle form, which usually ends in ed:

I have played the game.

Future Perfect is used to describe events or activities that will begin at some point in the future, before
another event or activity. The future perfect is formed by adding will have to the past participle form,
which usually ends in ed:

Michael is always late. By the time he arrives tonight, the party will have ended.

2. The progressive form indicates an action that is ongoing.

Past Progressive is used to describe an event or activity that was occurring when a second event or
activity occurred. The past progressive is formed by using was or were with the form of the verb that
ends in ing:

We were playing basketball when the car smashed through the gate.

Present Progressive is used to describe an event or activity that is happening currently: at the moment
the sentence is written. The present progressive is formed by adding am, is, or are to the form of the
verb that ends in ing:

We are eating dinner right now.

Future Progressive is used to describe an event or activity that will occur in the future, and will
continue to happen for an extended period of time. The future progressive is formed by adding will be or
shall be to the form of the verb that ends in ing:

For the next several months, Michelle will be traveling through Europe.

3. The perfect progressive form indicates an action that is ongoing, but will be completed at some
definite time.

I have been studying.

5-B. Verb Time Sequences: Verb Tense

Verb tenses exist in order to allow us to specify at what point in time some event occurred did it
happen at one point in the past, or is it still happening? Is it happening now, or will it happen in the
future? Because so many different tenses exist, GMAT questions are often extremely complicated, using
several different tenses in a single sentence. The correct tense (or tenses) makes the sequence of
actions clear.
Here's an example of a relatively simple verb tense error, and its correction:

Incorrect: After he had finished his performance, he would go to the party.

Correct: After he had finished his performance, he went to the party.

Why is the second sentence correct? Because the order of events is well clarified. Both events the
performance and the party happened in the past, but the performance happened first, and the party
second. Thus, both verbs should be in the past tense: had finished in past perfect, to indicate that the
performance happened first, and then went in simple past. The incorrect sentence implies that the
performance happened once in the past, but that his after-performance party attendance was ongoing
which doesn't make any sense.

Here's another example.

If the cyclist wins the race, it will be representing an extraordinary comeback from his earlier cancer.

To determine whether this sentence is correct, let's break it down into its constituent parts:

The "if clause" at the beginning of the sentence indicates a hypothetical: a sentence written in if...then...
form. This kind of sentence requires that the dependent event be in the simple future tense: meaning
that the event, if it happens, will happen once, at some time in the future, following the first event's
occurrence. It will not keep happening. Here, however, the dependent event is in the future continuous,
not the simple future.

Incorrect: If the cyclist wins the race, it will be representing an extraordinary comeback from his

earlier cancer.

Correct: If the cyclist wins the race, it will represent an extraordinary comeback from his earlier

cancer.

Why is the second sentence correct? Because a positive outcome of the race, which is as yet
undetermined, is only going to represent his comeback once as soon as it happens. The first sentence
implies that the cyclist's victory is going to keep representing a comeback for the duration of his victory
which is confusing, and doesn't make much sense.

To help determine whether the verbs in a sentence are in the proper tenses, pick one event as a "base"
action, and then try to figure out when other events occurred in relation to that event. Try to discern
whether the events occurred prior to the base action, or after the base action; or at the same time as
the base event took place. Keep in mind that actions that start before the base may continue after the
base.

Ask yourself: "What happened first, second? What makes sense logically?"

This is only half of the process: after you determine when the events took place, you still need to know
what verb form corresponds to the time sequence you've identified. This requires a working knowledge
of verb tense, as well as of mood and voice - it's very important to study them.

Tips for recognizing verb tense errors:

1. Watch for ing forms.


Typically, ing forms are used as junk answers on the GMAT; you will often be
given a better alternative.

I am walking

I was walking

I had been walking

2. Watch for time sequences.


Be alert for the appearance of several verbs indicating the occurrence of several
events that happen (or happened) at different points in time. Pick one verb as the
"base" in time sequence, and determine the order of events relative to the base
event.

5-C. Verb Time Sequences: Mood

In Modern English, "Mood" describes the manner in which the action or state expressed by a verb is
viewed with respect to its factuality, possibility, or command. Highly inflected languages, such as
German and Japanese, typically possess numerous possible moods. English, a weakly inflected
language, has only three basic classes of mood: Indicative, Imperative, and Subjunctive.
1.Indicative Mood states a fact or asks a question of fact. The indicative is, by far, the most commonly
used mood.

Harry spends all of his money on comic books.

Its simple enough: any statement related to a claim of fact is in the indicative mood. Mood is not
affected if it is unlikely Harry actually spends all of his money on comic books; all that matters is that the
verb usage is in the form of an assertion of fact.

How does Harry spend all of his money?

If we begin to answer with Harry spends all of his money on, we can see that the question is inquiring
into factuality in the same indicative sense as the previous sentence asserted factuality.

2. Imperative Mood expresses a command, request, or prohibition.

Spend all of your money on comic books!


Paul, please try to spend your money on something worthwhile.
Stop spending all of your money, boy.

Each sentence here illustrates a different shading of the imperative, but all of them issue a direct
address in the second person. The basic bluntness and singular addressing relegate the imperative
predominately to familiar speech and dialogue, but it can also be powerfully utilized in rhetoric ("Ask not
what your country can do for you...".).

3. The Subjunctive Mood is used chiefly to express the speakers attitude about the likelihood or
factuality of a given situation. It has a present and past form.

A. The present subjunctive is most familiar to us in stock phrases such as God help him, be that as it
may, come what may, and suffice it to say. It also occurs in "that clauses" used to state commands or to
express intentions or necessity:

We demand that Paul stop spending all of his money on comic books.
It is necessary that Paul stop this behavior.

B. The past subjunctive is sometimes called the "were subjunctive", since were is the only subjunctive
form that is distinct from the indicative past tense. It appears chiefly in
"if clauses" and in a few other constructions expressing hypothetical conditions:

If Paul were not spending all of his money, he would not be happy.
I wish Paul were not spending his money so recklessly.

These constructions might ring a bit funny and archaic to the ear, but, when used properly,
express subtleties of situation and condition that can sometimes be lost when the indicative is
used.

5-D. Verb Time Sequences: Voice

Transitive verbs possess a voice that describes the relationship between the subject and the
action of the verb.
Verbs in the active voice place the actor of the action in the subject role and the person or thing that is
acted upon sits in the object position.

These are sentences that contain verbs in the active voice:

Andy throws the baton.

The octogenarian plays the harp

The performers of the actionAndy and the octogenarian are the subjects of the sentence, and the
things acted uponthe baton and the harpare the objects.

The situation is reversed in the passive voice. The previous object that is acted upon becomes the
subject, and the actor finds himself in a prepositional phrase beginning with "by" or is excised from the
sentence altogether:

The baton was thrown (by Andy)

The harp was played (by the octogenarian).

Regular use of the passive voice without a specific contextual cause is awkward, wordy, and
unnecessarily, well, passive.

5-E. Verb Time Sequences: Sample Questions

1. Valerie recalls her college years with such nostalgia that she often lost herself in reminiscence.

A. she often lost herself in reminiscence.


B. she often had lost herself in reminiscence.
C. she often loses herself in reminiscence.
D. she often will be losing herself in reminiscence.
E. she often will have lost herself in reminiscence.

Valerie recalls her college years with such nostalgia that she often lost herself in reminiscence.

Grammar issue presented: verb tense


recalls sets up a situation in the present. Though Valiere's college years took place in the past, she
remembers them now, and therefore loses herself in reminiscence (present tense).
A. she often lost herself in reminiscence.
Proper tense? NO (past)

B. she often had lost herself in reminiscence.


Proper tense? NO (past-perfect)

C. she often loses herself in reminiscence.


Proper tense? YES (present)

D. she often will be losing herself in reminiscence.


Proper tense? NO (future)

E. she often will have lost herself in reminiscence.


Proper tense? NO (future perfect)

(C) is the only choice which uses the correct tense, remaining consistent with the situation that is
described in the non-underlined portion of the sentence.

2. Before it will be made available to the public, the painting was sold to a private collector and is
currently on hold at the auction house awaiting pick-up.

A. Before it will be made available to the public


B. Before it can be made available to the public
C. Before it could have been made available to the public
D. Before it has been made available to the public
E. Before it was made available to the public

Before it will be made available to the public, the painting was sold to a private collector and is currently
on hold at the auction house awaiting pick-up.

Grammar issue presented: verb tense


The underlined portion of the sentence is in the wrong tense. We are setting up a hypothetical situation,
saying that the painting could not be made available to the public, because it was sold to a private
collector before the public was able to gain access to it. We need to use the future perfect.

The correct sequence of events is this:


1) Sold to a private collector
2) Hypothetically: would have been available to the public
3) On hold at auction house

A. Before it will be made available to the public


Proper tense? NO (future progressive)
B. Before it can be made available to the public
Proper tense? NO (present)

C. Before it could have been made available to the public


Proper tense? YES (past progressive)

D. Before it has been made available to the public


Proper tense? NO (past)

E. Before it was made available to the public


Proper tense? NO (past)

(C) is the only choice which uses the correct tense. It sets up a hypothetical situation in the past, from
which the other events in the sentence end up taking the place of (also in the past).

3. In 79 CE, when Vesuvius erupted, the Villa of the Papyri was being covered in 90 feet of volcanic
ash, and was preserved as the sole library of Antiquity.

A. was being covered in 90 feet of volcanic ash, and was preserved


B. was covered in 90 feet of volcanic ash, and was preserved
C. was covered in 90 feet of volcanic ash, and had been preserved
D. had been covered in 90 feet of volcanic ash, and is preserved
E. is covered in 90 feet of volcanic ash, and was preserved

In 79 CE, when Vesuvius erupted, the Villa of the Papyri was being covered in 90 feet of volcanic ash,
and was preserved as the sole library of Antiquity.

Grammar issue presented: verb tense


The beginning of the sentence sets up a situation: when Vesuvius erupted. This means that the rest of
the sentence is going to tell us what happened during the time of the eruption. Since the eruption
happened in the past, the rest of the sentence will have to be in the past as well.

A. was being covered in 90 feet of volcanic ash, and was preserved


Proper tense? NO (past progressive): was being, the past progressive, does not make sense with the
rest of the sentence: was preserved would have to also be in the past progressive for this to work

B. was covered in 90 feet of volcanic ash, and was preserved


Proper tense? YES (past)
Additional errors? NONE

C. was covered in 90 feet of volcanic ash, and had been preserved


Proper tense? YES (past)
Additional errors? YES 'had been preserved' (past perfect)
D. had been covered in 90 feet of volcanic ash, and is preserved
Proper tense? NO (past-perfect progressive)
Additional errors? YES is preserved (present)

E. is covered in 90 feet of volcanic ash, and was preserved


Proper tense? NO (present)

(B) correctly contains the past tense throughout.

4. In the history of life on earth, there have been five mass extinctions; the most recent is the K-T
extinction, which occurred at the end of the Cretaceous period and wiped out the non-avian dinosaurs,
among many other species.

A. the K-T extinction, which occurred at the end of the Cretaceous period and wiped out
B. the K-T extinction, which had occurred at the end of the Cretaceous period and wiped out
C. the K-T extinction, which, occurring at the end of the Cretaceous period had wiped out
D. the K-T extinction which, began occurring at the end of the Cretaceous period and wiped out
E. the K-T extinction which, occurs at the end of the Cretaceous period and wiped out

In the history of life on earth, there have been five mass extinctions; the most recent is the K-T
extinction, which occurred at the end of the Cretaceous period and wiped out the non-avian dinosaurs,
among many other species.

Grammar issue presented: verb tense


The beginning of the sentence tells us that there have been five mass extinctions. Because the
sentence uses have been, the present perfect progressive tense, describing an action that began in the
past and continues in the present, we know that mass extinctions are ongoing phenomena. However,
the rest of the sentence describes extinctions of the past, and so, must also be in the past.

A. the K-T extinction, which occurred at the end of the Cretaceous period and wiped out
Proper tense? YES (past)

B. the K-T extinction, which had occurred at the end of the Cretaceous period and wiped out
Proper tense? NO (past-perfect, past): Since we are only discussing one event: the K-T extinction, the
past-perfect is unnecessary. Using the past perfect makes the details of the K-T extinction seem to be
separate events from the extinction itself.

C. the K-T extinction, which, occurring at the end of the Cretaceous period, had wiped out
Proper tense? NO (present, past-perfect)
Additional errors: Run-on!

D. the K-T extinction, which began occurring at the end of the Cretaceous period and wiped out
Proper tense? YES (past)
Additional errors: Unclear! The extinction happened long ago, so it is not necessary to state when it
began or that it continued on. Doing so makes it unclear whether the event had a definite end or if it is
still in progress.

E. the K-T extinction, which occurs at the end of the Cretaceous period and wiped out
Proper tense? NO (present, past)

(A) uses the past tense throughout the description of the K-T extinction. It is the most clear, and
therefore, the best choice.

6. Comparisons: Introduction

Comparisons

A. Introduction
B. Comparisons as Parallelism
C. Comparative and Superlative Forms
D. Sample Questions

You should only compare things that can be logically compared. Faulty or nonsensical
comparisons account for a significant number of the errors in GMAT Sentence Correction
questions. Most of these errors derive from a very simple concept: you can't compare apples to
oranges. You are entirely welcome, however, to compare apples to apples, or a long sweater to a long
coat, or even the baking of apple turnovers to the baking of pineapple turnovers. That is, on the GMAT,
you want to compare only those things that are grammatically or logically similar. For instance, you can't
logically compare a person ("Joe") to a quality ("purple"), or an item ("a banana") to a group ("the
NYPD"). You have to compare one individual to another individual, one quality to another quality, or one
group to another group.

Often, a comparison will sound as though it's acceptable, but will be missing a few necessary
words:

Incorrect: The view from this apartment is not nearly as spectacular as from that mountain lodge.

If you read it quickly, this sentence makes perfect sense: the view from the apartment is being compared
to the view from the mountain lodge. But if you look more closely, you'll see that the sentence actually
compares the view from the apartment to something about the lodge but what about it?

The comparison needs to be clarified.


Just like misplaced modifier questions, comparison questions can't be judged by the ear alone: even
though you might understand what the writer is trying to say, trying doesn't cut it on the GMAT. You have
to make sure the sentence actually says what it means to say. Here's the correct version:

The insertion of two little words - "the one" - makes this sentence grammatically correct, because "the
view from" now has a partner in comparison: "the one from." An alternative would be to repeat "the view
(from)," instead of "the one (from)," in the latter portion.

Incorrect: The view from this apartment is not nearly as spectacular as from that mountain lodge.

Correct: The view from this apartment is not nearly as spectacular as the one from that mountain

lodge.

Correct: The view from this apartment is not nearly as spectacular as the view from that mountain

lodge.

Let's look at another example.

Shakespeare's plays are different from any other playwrights of his era because they exhibit an
exceptional mastery of verse.

Once again, the sentence sounds ok, but it actually compares Shakespeare's plays to other playwrights.
The comparison should be between his plays and the plays of other writers.

How can we fix it? We can make the sentence reasonable by inserting a few choice words that clarify
the nature of the comparison:

Like the phrase "the one from" in the last example, the phrase "those of" in this example makes it very
clear that Shakespeare's plays are being compared to other playwrights' plays not other playwrights.
Incorrect: Shakespeare's plays are different from any other playwrights of his era because they

exhibit an exceptional mastery of verse.

Correct: Shakespeare's plays are different from those of any other playwrights of his era

because they exhibit an exceptional mastery of verse.

You should look out for key comparison words, such as:

like as compared to

less than more than other

that of those of

Comparison, as a concept, is closely related to parallelism. The basic idea theme is that you should
always make perfectly clear to the reader the entities being compared.

Some common phrases used in comparisons are as much as, more than, less than, just like, as, and
that of.

6-B. Comparisons: Comparisons as Parallelism

Comparisons are a special case of parallelism. A number of comparison-specific constructions call


for you to always express ideas in parallel form. These constructions include:

Either X or Y...

Neither X nor Y...

Not only X but also Y...

X or Y can stand for as little as one word, or as much as an entire clause, but in every case, the
grammatical structure of X or Y must be identical. For example, the sentence Either drinking or to eat
will do violates the rule by mismatching verb forms:
This is a comparison, and requires parallelism. Both verbs must be in the same form: because they
aren't currently in the same form, one must be adjusted.

Both verbs are now in the ing form. Though in many cases of parallelism either verb form is fine, for
Either/Or comparisons such as this one, both verbs must be in the ing form.

Here's another example, using Neither/Nor:

Neither an interest in history nor to be adept in a foreign language is going to help you learn to sing.

This sentence lists two talents one could possess, in a neither/or format. They are not, however, in the
same form.

In this sentence, a noun is compared to a verb. Though it's a different kind of mistake than the missing-
information and verb-form errors we've looked at, it should be dealt with in the same way: by changing
one of the forms to match the other.

Both phrases are now in the same form: "an interest in" and "an adeptness in". In this instance, the verb
had to be changed to match the noun, instead of the other way around, because "to be" verbs don't
belong in comparison (either/or, neither/nor) sentences.

Incorrect: Neither an interest in history nor to be adept in a foreign language is going to help you

learn to sing.

Correct: Neither an interest in history nor an adeptness in a foreign language is going to help

you learn to sing.


6-C. Comparisons: Comparative and Superlative Forms

Some comparison words are just special forms of adjectives; instead of describing one thing,
they describe the relationship between two or more things. For most adjectives, use the following
guidelines when forming or using comparatives or superlatives:

Comparative:
used when comparing two things

usually formed by adding "er" to the end of the word: happier, softer, faster, taller

Superlative:
used when comparing more than two things

usually formed by adding "est" to the end of the word: happiest, softest, fastest, tallest

Special rules apply for irregular forms. Below is a list of those adjectives which have irregular
comparative forms; beneath each is listed its comparative and superlative form.

good many
better / best more / most

bad little
worse / worst little, lesser, less / least

much far
more / most farther, further / farthest, furthest

6-D. Comparisons: Sample Questions

EASY

1. Even though he does not like crowds, John still likes New York City more than Sybil.
a) John still likes New York City more than Sybil.
b) New York City is still liked more than Sybil by John.
c) John is still liking New York City more than is Sybil.
d) John still likes New York City more than Sybil does.
e) New York City is still more liked by John than Sybil.

This question tests comparison. (A) is improperly compared because it compares how much John likes
New York City to how much he likes Sybil, NOT to how much Sybil likes New York City. (B) and (E)
moves around the words, but still maintain improper comparisons. (C) uses is liking which is
grammatically incorrect. (D) is the only choice which compares Johns affinity for New York City to
Sybils affinity for New York City, without adding new errors.

HARD

2. In his work, George Santayana is more reminiscent of Plato's poetic narratives and Henry David
Thoreau's obsessive detailing than Bertrand Russells scientific precisions.

a) In his work, George Santayana is more reminiscent of


b) George Santayana writes more like
c) George Santayana reminds one more of
d) George Santayana's work is more reminiscent of
e) George Santayanas work more resembles that of

This question tests comparison. Because the sentence compares the styles of several writers, it must
be made clear that Santayana's work is the subject of the sentence, not Santayana himself. Choice C,
omitting any mention of 'work', is therefore incorrect. Though choices A and B do mention work, they fail
to do so in a manner that maintains parallelism with the non-underlined part of the sentence, which
refers to Plato's narratives and Thoreau's detailing. Choice D makes incorrect use of the word
'reminiscent', which is not generally used in literary comparisons. Choice E maintains parallelism and
uses the word 'resembles', which is preferable to 'reminiscent', and is therefore the best choice.

7. Idioms: Introduction

Idioms

A. Introduction
B. List of Idioms
C. Sample Questions

Idioms are not hard and fast rules of grammar. Instead, they're verbal habits and preferences that
have become ingrained in the English language after many years of repeated use. To prepare for idiom
questions, take a look at the list of common idioms below, split them into two lists those you know, and
those you don't know and memorize the ones you don't know. It can also help to start reading every
day, as idioms appear in almost every kind of reading material available.

Look for these common tricks on GMAT questions:

Consider, regard... as, think of...as: there is no as after consider, while both regard and
think of need the as.

To be/being: In general, avoid the construction to be/being because they are usually
passive. To be/being are commonly used in junk answer choices.

7. Idioms: List of Idioms

Idioms in Bold tend to be more common on the GMAT.

access to The company has access to large capital reserves.

act as The poison pill in the contract acts as a preventative measure


against hostile takeovers.

acclaimed as He was acclaimed as a brilliant director.

allows for The design of the robot arm allows for great flexibility.

as...as Chocolate tastes as good as ice cream.

associate with He associates beer with potato chips.

attribute to The poor first quarter results are attributed to the


restructuring.

a responsibility to The CEO has a fiduciary responsibility to all shareholders.

a result of The recent Nasdaq decline is a result of higher interest rates.

a sequence of The Sumerian text was a sequence of incomprehensible


symbols.

agree with The Teamsters do not agree with the Republicans on many
issues.

among Used when discussing more than two items. He was the
finest policeman among the hundreds of rookies.
as good as/or better The new software is as good as or better than anything on
than the market.

as great as The new house looks as great as I had hoped.

attend to (someone) The emergency room doctor attended to the injured victim.

attribute X to Y/X We attribute the results to the new management.

attributed to Y The extinction of the dinosaurs has been attributed to an


asteroid collision.

based on The results are based on a comprehensive ten year study.

begin to He will begin to study twelve hours before the test.

believe X to be Y After seeing the flying saucer, I believe UFOs to be a real


phenomenon

between Used when discussing two things (if there are more than two,
then use among instead). He could not decide between Corn
Flakes and Raisin Bran.

care about How much do business schools care about your score?

centers on + noun The GMAT centers on the knowledge of basic math and
writing/reading skills.

choose to The number of students who choose to go to business school


has increased in the last ten years.

consistent with Your grades are not consistent with your abysmal GMAT
scores.

contend that He contends that the GMAT has a cultural bias.

consider + noun How important do you consider the test?

continue + to If you continue to study, you will succeed.

contrast A with B If you contrast A with B, you can see the difference.

convert to You may convert muscle to fat if you study too much.
compare A to B Compare to stresses similarities. The music critic favorably
compared him to Bob Dylan.

compare A with B Compare with stresses differences. Broccoli is good for you
compared with ice cream.

count on + noun He counts on management support.

concerned with They are concerned with investor relations more than actual
profitability.

conform to When you work at a new company, you should try to conform
to its corporate culture.

decide to We decided to continue.

decide on We decided on the new format.

depend on The global economy depends on improving productivity.

different from The CAT is very different from the paper and pencil GMAT.

difficult to Many students find the CAT difficult to take.

distinguish between X Distinguish between domestic and international production.


and Y

distinguish X from Y Juries must attempt to distinguish truth from falsehood.

depends on whether Our place in the playoffs depends on whether we win tonight.

to be + essential to + Speed is essential to success in the Internet marketplace.


noun

except for He did well on the GMAT, except for the sentence correction
questions.

flee from The convict fled from the country.


G

grow from Dell Computer grew from a start-up to a Fortune 500


company in less than fifteen years.

grow out of Needless to say, they quickly grew out of their first office.

help + noun + to Their direct business model helped them to grow rapidly.

indicate that Dell's recent stock trouble may indicate that their growth will
not continue to be as rapid.

invest in He is too risk-averse to invest in the stock market.

identical with His DNA is identical with his twin's.

in contrast to The candidate claims to support tax cuts, in contrast to his


prior statements.

independent from The Federal Reserve Board is supposed to be independent


from political considerations.

indifferent towards Some countries are indifferent towards human rights.

leads to Rapid growth often leads to problems.

like Usually used only for direct comparison: He walks like Joe
walks.

localized in Most Internet venture capital is localized in a few areas of the


world.

mistook + noun + for I mistook you for an old friend.

modeled after The judicial building is modeled after the Parthenon.

more than ever Companies demand MBA graduates now more than ever.
N

native to There is a unique business culture native to the U.S.

a native of It infects those who are not even a native of America.

need to Living in New York City is an experience everyone needs to


try.

to be + necessary + to It is necessary to get a high GMAT score to get into Stanford.

neither...nor Neither Tom nor Sam has the necessary skills to finish the
job.

not only...but also Stanford not only has the highest GMAT average, but also
the highest GPA.

prohibit from + gerund You are prohibited from using a calculator on test day.

potential to A graduate of a top business school has the potential to


make over $150,000.

range from X to Y The GMAT scores at top business schools will range from
650 to 750.

refer to If you have any more questions, you should refer to a


grammar book.

regard as Wharton's finance program is regarded as the finest in the


world.

require + noun + to You require a GMAT score to go to most U.S. business


schools.

rivalry between X and Y The rivalry between the Boston Red Sox and the New York
Yankees is one of the most celebrated in professional sports.

responsible for The manager is responsible for seven entry level employees.

retroactive to The tax policy change is retroactive to last year.


S

save for Save for William, no one else passed the exam.

save from Many people use business school to save them from dull
jobs.

so that So should not be used as an adjective: GMAT preparation is


so... boring. Use it with "that." This guide is designed so that
you may raise your score.

subscribe to Business school students should subscribe to the Wall Street


Journal.

tie to The contract should be tied to concessions.

transmit to The communications system will transmit to anyone within


range.

used + infinitive Japan used to be the model industrial economy.

to be + used to + After five practice tests, he was used to the GMAT CAT
gerund format.

Once again - the most effective way to learn idioms is to practice them. Whenever you get an
idiom question wrong, write down the idiom. There are a finite number of idioms that could be tested on
the GMAT, and with enough practice, you should be able to cover most of them.

Examples

1. When choosing a car you often have to choose (between/among) practicality and performance."
"Between" is correct. Use "between" to distinguish two things, such as "practicality" and "performance."
Use "among" for more than two things: "The five bank robbers divided the stolen money among
themselves."

2. A small order of french fries has (fewer/less) fries than the super-sized order.
"Fewer" is correct. "Fewer" answers the question "How many?", while "less" answers the question "how
much?". That is, "fewer" refers to things that can be counted (birds, airplanes, french fries, blades of
grass), and "less" refers to things that can't be counted individually, and are usually referred to en
masse, such as pudding, water, or flour.

3. I prefer Mozart (to/over) Beethoven.


"Preferto" is the proper expression.
4. Timothy talks (like/as) his friends do.
This is one of the few instances where "like" should be used in English. "Like" is used here as a direct
comparison.

5. He was studying (in/at) a rate of two practice GMAT's per day.


The proper expression is "at a rate of," not "in a rate of."

6. The joint-venture contract covers such questions (like/as) the division of profits and costs.
"Covers as" is better here. "Like" should be used very rarely, only for direct comparisons (Joe looks
like his brother).

7. Dan Marino is regarded (as/to be) one of the greatest quarterbacks to ever play football.
The proper idiom in this example is "regarded as."

7. Idioms: Sample Questions

EASY

1. Sheila is a successful business pioneer, who attempted at creating her own company at a very young
age.

a) who attempted at creating


b) and she attempted at creating
c) who has attempted to be creating
d) who attempted to create
e) and she attempted to be creating

Explanation: This question tests idiom usage. Because the proper expression is attempt to, (A),(B),
and (C) are incorrect. With the use of has attempted to be creating, choice (E) is wordy, and a confusing
mix of past and present tenses. (D) is grammatically correct and is also the most concise way to convey
the sentences meaning.

HARD

2. In 1904, the Abbey Theatre of Dublin, Ireland, was founded by theater supporters who were
concerned at reviving appreciation for Irish culture through playwriting and literature.

a) was founded by theater supporters who were concerned at


b) became founded by theater supporters who were concerned at
c) was founded by theater supporters who were concerned with
d) had been founded by theater supporters who had to become concerned with
e) was founded by theater supporters who have concerns about

Explanation: This question tests idiom, as well as tense and wordiness. (A) contains the incorrect
concerned at. The verb concerned takes with or about. Therefore, (B) is also incorrect. (D) is confusing.
Had been founded is unnecessarily in the past perfect, while had to become indicates that the founders
were forced into feeling concerned, changing the meaning of the sentence. (E) uses the present have
concerns to continue a description of people from the past. Only (C) is in the past tense throughout, is
clear in meaning and uses a proper idiom.

NOTE: There is a set of sentence correction sample chapter. These are timed and intended to be taken
online to best simulate the actual GMAT. Sample Questions

Vous aimerez peut-être aussi